You are on page 1of 87

2010

SANKET
SHAH
(KESAR SAL)
Available on www.mysarthee.co.cc
NEUROLO!
NORMAL CSF
Constituent Conventional Units
l"cose #$%&$ m'()L
Lactate *$%+$ m'()L
Total ,rotein
L"mbar *-%-$ m'()L
.isternal *-%+- m'()L
/entric"lar 0%*- m'()L
Alb"min 0.0%##.+ m'()L
1' $.2%-.& m'()L
1' in)e3
b

$.+2%$.-2
Oli'oclonal ban)s
(O4)
5+ ban)s not ,resent in
matche) ser"m sam,le
Ammonia +-%6$ '()L
.S7 ,ress"re -$%*6$ mmH
+
O

.S7 vol"me (a)"lt) 8*-$ mL
Re) bloo) cells $
Le"9ocytes
Total $%- monon"clear cells ,er
mm
:

;i<<erential
Lym,hocytes 0$%&$=
>onocytes :$%-$=
Ne"tro,hils None
a
Since cerebros,inal <l"i) concentrations are e?"ilibri"m val"es@
meas"rements o< the same ,arameters in bloo) ,lasma
obtaine) at the same time are recommen)e). However@ there is
a time la' in attainment o< e?"ilibri"m@ an) cerebros,inal levels
o< ,lasma constit"ents that can <l"ct"ate ra,i)ly (s"ch as
,lasma 'l"cose) may not achieve stable val"es "ntil a<ter a
si'ni<icant la' ,hase.
b
1' in)e3 A .S7 1'(m'()L) 3 ser"m alb"min('()L)(Ser"m
1'('()L) 3 .S7 alb"min(m'()L).
CSF IN BACTERIAL
MENINGITIS
The diagnosis of bacterial meningitis is made by
e3amination o< the .S7
In an immunocompetent patient with
*.no 9nown history o< recent hea) tra"ma@
+. a normal level o< conscio"sness@ an)
:.no evi)ence o< ,a,ille)ema or <ocal ne"rolo'ic
)e<icits@
it is consi)ere) sa<e to ,er<orm LB witho"t ,rior
ne"roima'in' st")ies.
If LP is delayed in order to obtain neuroimaging
st")ies@ em,irical antibiotic thera,y sho"l) be initiate) a<ter
bloo) c"lt"res are obtaine).
* O,enin'
,ress"re
C*6$ mmH
+
O
(2$=)

+ Dhite bloo)
cells
*$("L to *$@$$$("LE ne"tro,hils ,re)ominate
(2$=)
: Re) bloo) Absent in nontra"matic ta,
cells
# l"cose 5#$ m'(;l
- .S7(ser"m
'l"cose
5$.#
(0$=)
0 Brotein C#- m'()L
(2$=)
& ramFs stain Bositive in
(C0$=)
6 ."lt"re Bositive in
(C6$=)
2 Late3
a''l"tinatio
n
>ay be ,ositive in ,atients with menin'itis )"e
to S. pneumoniae@ N. meningitidis@ H.
influenzae ty,e b@ E. coli@ 'ro", 4 stre,tococci
*$ Lim"l"s
lysate
Bositive in cases o< 'ramGne'ative menin'itis
** B.R ;etects bacterial ;NA
.
e
r
e
b
r
o
s
,
i
n
a
l

7
l
"
i
)

(
.
S
7
)

A
b
n
o
r
m
a
l
i
t
i
e
s

i
n

4
a
c
t
e
r
i
a
l

>
e
n
i
n
'
i
t
i
s
Use of the CSFserum glucose ratio corrects <or
hy,er'lycaemia that may mas9 a relative )ecrease in the
.S7 'l"cose concentration.
!he CSF glucose concentration is low when the
.S7(ser"m 'l"cose ratio is 5$.0.
A .S7(ser"m 'l"cose ratio 5$.# is hi'hly s"''estive o<
bacterial menin'itis b"t may also be seen in other
conditions@ including fungal, tuberculous, and carcinomatous
meningitis.
The Limulus amebocyte lysate assay is a ra,i) )ia'nostic
test <or the )etection o< 'ramGne'ative en)oto3in in .S7 an)
th"s <or ma9in' a )ia'nosis o< 'ramGne'ative bacterial
menin'itis.
The test has a specificity of "#$100% and a sensitivity
approaching 100%& Th"s@ a ,ositive Lim"l"s amebocyte
lysate assay occ"rs in virt"ally all ,atients with 'ramG
ne'ative bacterial menin'itis@ b"t <alse ,ositives may occ"r.

1& I'!()
Ac"te in<ections o< the nervo"s system are amon' the most
im,ortant ,roblems in me)icine beca"se early reco'nition@ e<<icient
)ecisionGma9in'@ an) ra,i) instit"tion o< thera,y can be li<esavin'.
2& *+FI'I!I)'
Bacterial meningitis is an Ac"te ,"r"lent in<ection within the
s"barachnoi) s,ace.
1t is associate) with a .NS in<lammatory reaction that may
res"lt in )ecrease) conscio"sness@ seiH"res@ raise) intracranial
,ress"re (1.B)@ an) stro9e.
,& +PI*+-I)L)./
."rrently@ the or'anisms most commonly res,onsible <or comm"nityG
ac?"ire) bacterial menin'itis are
Streptococcus pneumoniae (8-$=)@
N. meningitidis (8+-=)@
ro", 4 stre,tococci (8*-=)@
Listeria monocytogenes (8*$=).
H. influenzae now acco"nts <or 5*$= o< cases o< bacterial
menin'itis in most series.
0& +!I)L)./
S. pneumoniae
the most common ca"se o< menin'itis in a)"lts C+$ years o<
a'e
Bre)is,osin' <actors
*) ,ne"mococcal ,ne"monia.
+) coe3istin' ac"te or chronic ,ne"mococcal sin"sitis or
otitis me)ia@
:) alcoholism@
#) )iabetes@
-) s,lenectomy@
0) hy,o'amma'lob"linemia@
&) com,lement )e<iciency@
6) hea) tra"ma with basilar s9"ll <ract"re an) .S7
rhinorrhea.

N. Meningitides
chil)ren an) yo"n' a)"lts between the a'es o< + an) +$.
The ris9 o< invasive )isease <ollowin' naso,haryn'eal coloniHation
)e,en)s on both bacterial vir"lence <actors an) host imm"ne
)e<ense mechanisms
1n)ivi)"als with )e<iciencies o< any o< the com,lement
com,onents@ incl")in' ,ro,er)in@ are hi'hly s"sce,tible to
menin'ococcal in<ections.
Enteric 'ramGne'ative bacilli
common ca"se o< menin'itis in in)ivi)"als with chronic an)
)ebilitatin' )iseases s"ch as )iabetes@ cirrhosis@ or alcoholism an)
in those with chronic "rinary tract in<ections
ramGne'ative menin'itis can also com,licate ne"ros"r'ical
,roce)"res@ ,artic"larly craniotomy.
ro", 4 stre,tococc"s@
,revio"sly res,onsible <or menin'itis ,re)ominantly in neonates@
b"t it has been re,orte) with increasin' <re?"ency in in)ivi)"als
C-$ years o< a'e@ ,artic"larly those with "n)erlyin' )isease
L. Monocytogenes
im,ortant ca"se o< menin'itis in neonates (5* month o< a'e)@
,re'nant women@ in)ivi)"als C0$ years@ an)
imm"nocom,romise) in)ivi)"als o< all a'es.
7oo)borne h"man listerial in<ection
H. influenzae ty,e b
menin'itis in chil)ren has )ecline) )ramatically since the
intro)"ction o< the Hib conI"'ate vaccine
>ore <re?"ently@ H. influenzae ca"ses menin'itis in "nvaccinate)
chil)ren an) a)"lts.
Staphylococcus aureus an) coa'"laseGne'ative sta,hylococci
im,ortant ca"ses o< menin'itis that occ"rs <ollowin' invasive
ne"ros"r'ical ,roce)"res@ ,artic"larly sh"ntin' ,roce)"res <or
hy)roce,hal"s
#& P1!2)P2/SI)L)./
RE77ER HARR1SON 71URE :&0G+
3& CLI'IC1L P(+S+'!1!I)'S
BRESENTAT1ON
1) an ac"te <"lminant illness that ,ro'resses ra,i)ly in a <ew ho"rs
OR
+) s"bac"te in<ection that ,ro'ressively worsens over several
)ays
.LASS1.AL TR1A;
*) <ever@
+) hea)ache@
:) n"chal ri'i)ity
OTHER 1>BORTANT
*) )ecrease) level o< conscio"sness
+) Na"sea@
:) ,roIectile vomitin'@
#) ,hoto,hobia
SeiH"res
*) 7ocal
+) eneralise)
RA1SE; 1.B
*) maIor ca"se o< obt"n)ation an) coma
2) 2$= o< ,atients will have a .S7 o,enin' ,ress"re C*6$ mmH
+
O@
an) +$= have o,enin' ,ress"res C#$$ mmH
+
O.
:) )eterioratin' or re)"ce) level o< conscio"sness@
#) ,a,ille)ema@
-) )ilate) ,oorly reactive ,",ils@
0) si3th nerve ,alsies@
&) )ecerebrate ,ost"rin'@
6) ."shin' re<le3 (bra)ycar)ia@ hy,ertension@ an) irre'"lar
res,irations).
2) .ERE4RAL HERN1AT1ON as a com,lication
S,eci<ic clinical <eat"res
*) rash o< menin'ococcemia@ which be'ins as a )i<<"se
erythemato"s mac"lo,a,"lar rash resemblin' a viral e3anthemE
however@ the s9in lesions o< menin'ococcemia ra,i)ly become
,etechial. Betechiae are <o"n) on the tr"n9 an) lower e3tremities@
in the m"co"s membranes an) conI"nctiva@ an) occasionally on
the ,alms an) soles
.
+) R1.KETT1S1AL RASHGcharacteristic rash within 20 h o< the onset
o< sym,toms. The rash is initially a )i<<"se erythemato"s
mac"lo,a,"lar rash that may be )i<<ic"lt to )istin'"ish <rom that
o< menin'ococcemia. 1t ,ro'resses to a ,etechial rash@ then to a
,"r,"ric rash an)@ i< "ntreate)@ to s9in necrosis or 'an'rene. The
color o< the lesions chan'es <rom bri'ht re) to very )ar9 re)@ then
yellowishG'reen to blac9. The rash ty,ically be'ins in the wrist an)
an9les an) then s,rea)s )istally an) ,ro3imally within a matter o<
a <ew ho"rs@ involvin' the ,alms an) soles
4& *I1.')SIS
CSF
The diagnosis of bacterial meningitis is made by
e3amination o< the .S7
In an immunocompetent patient with
#.no 9nown history o< recent hea) tra"ma@
-. a normal level o< conscio"sness@ an)
0.no evi)ence o< ,a,ille)ema or <ocal ne"rolo'ic
)e<icits@
it is consi)ere) sa<e to ,er<orm LB witho"t ,rior
ne"roima'in' st")ies.
If LP is delayed in order to obtain neuroimaging
st")ies@ em,irical antibiotic thera,y sho"l) be initiate) a<ter
bloo) c"lt"res are obtaine).
* O,enin'
,ress"re
C*6$ mmH
+
O
(2$=)

+ Dhite bloo)
cells
*$("L to *$@$$$("LE ne"tro,hils ,re)ominate
(2$=)
: Re) bloo)
cells
Absent in nontra"matic ta,
# l"cose 5#$ m'(;l
- .S7(ser"m
'l"cose
5$.#
(0$=)
0 Brotein C#- m'()L
(2$=)
& ramFs stain Bositive in
(C0$=)
6 ."lt"re Bositive in
(C6$=)
2 Late3
a''l"tinatio
n
>ay be ,ositive in ,atients with menin'itis )"e
to S. pneumoniae@ N. meningitidis@ H.
influenzae ty,e b@ E. coli@ 'ro", 4 stre,tococci
*$ Lim"l"s
lysate
Bositive in cases o< 'ramGne'ative menin'itis
** B.R ;etects bacterial ;NA
.
e
r
e
b
r
o
s
,
i
n
a
l

7
l
"
i
)

(
.
S
7
)

A
b
n
o
r
m
a
l
i
t
i
e
s

i
n

4
a
c
t
e
r
i
a
l

>
e
n
i
n
'
i
t
i
s
Use of the CSFserum glucose ratio corrects <or
hy,er'lycaemia that may mas9 a relative )ecrease in the
.S7 'l"cose concentration.
!he CSF glucose concentration is low when the
.S7(ser"m 'l"cose ratio is 5$.0.
A .S7(ser"m 'l"cose ratio 5$.# is hi'hly s"''estive o<
bacterial menin'itis b"t may also be seen in other
conditions@ including fungal, tuberculous, and carcinomatous
meningitis.
The Limulus amebocyte lysate assay is a ra,i) )ia'nostic
test <or the )etection o< 'ramGne'ative en)oto3in in .S7 an)
th"s <or ma9in' a )ia'nosis o< 'ramGne'ative bacterial
menin'itis.
The test has a specificity of "#$100% and a sensitivity
approaching 100%& Th"s@ a ,ositive Lim"l"s amebocyte
lysate assay occ"rs in virt"ally all ,atients with 'ramG
ne'ative bacterial menin'itis@ b"t <alse ,ositives may occ"r.
'+U() I-1.I''I'. S!U*/
>R1 is ,re<erre) over .T beca"se o< its s",eriority in
)emonstratin' areas o< cerebral e)ema an) ischemia.
)i<<"se menin'eal enhancement is o<ten seen a<ter the
a)ministration o< 'a)olini"m
5I)PS/
Betechial s9in lesions@ i< ,resent@ sho"l) be bio,sie). The rash o<
menin'ococcemia res"lts <rom the )ermal see)in' o< or'anisms
with vasc"lar en)othelial )ama'e@ an) bio,sy may reveal the
or'anism on ramFs stain.
"& *IFF+(+'!I1L *I1.')SIS
*. /iral menin'oence,halitis@ an) ,artic"larly her,es sim,le3
vir"s (HS/) ence,halitis
+. Ric9ettsial )isease can resemble bacterial menin'itis
:. Ehrlichioses
#. 7ocal s",,"rative .NS in<ections @ incl")in' s"b)"ral an)
e,i)"ral em,yema an) brain abscess
-. S"barachnoi) hemorrha'e
0. chemical menin'itis )"e to r",t"re o< t"mor contents into the
.S7
&. sarcoi)@ systemic l","s erythematos"s (SLE)@ an) 4ehJetFs
syn)rome
6& !(+1!-+'!
.)1L
4acterial menin'itis is a me)ical emer'ency.
The 'oal is to be'in antibiotic thera,y within 0$ min o< a ,atientFs
arrival in the emer'ency room
+mpirical antimicrobial therapy
initiate) in ,atients with s"s,ecte) bacterial menin'itis be<ore the
res"lts o< .S7 ramFs stain an) c"lt"re are 9nown
Antibiotics Use) in Em,irical Thera,y o< 4acterial >enin'itis
an) 7ocal .NS 1n<ections
a
Indication 1ntibiotic
*. Breterm in<ants
to in<ants 5*
month
Am,icillin K .e<ota3ime
+. 1n<ants *%:
mos
Am,icillin K .e<ota3ime or
ce<tria3one
:. 1mm"nocom,e
tent chil)ren C:
mos an) a)"lts
5--
vancomycin K .e<ota3ime or
ce<tria3one
#. A)"lts C-- an)
a)"lts o< any a'e
with alcoholism or
other )ebilitatin'
illnesses
Am,icillin K ce<ota3ime or ce<tria3one
K vancomycin
-. Hos,italG
ac?"ire)
menin'itis@
,osttra"matic or
,ostne"ros"r'ery
menin'itis@
ne"tro,enic
,atients@ or
,atients with
im,aire) cellG
me)iate)
imm"nity
Am,icillin K ce<taHi)ime K
vancomycin
!otal *aily *ose and *osing Interval
1ntimicr
obial
1gent
Child 781 month9 1dult
Am,icillin +$$ (m'(9')()@ ?#h *+ '()@ ?#h
.e<ota3im
e
+$$ (m'(9')()@ ?0h *+ '()@ ?0h
.e<tria3on
e
*$$ (m'(9')()@ ?*+h # '()@ ?*+h
.e<taHi)i *-$ (m'(9')()@ ?6h 0 '()@ ?6h
Antibiotics Use) in Em,irical Thera,y o< 4acterial >enin'itis
an) 7ocal .NS 1n<ections
a
Indication 1ntibiotic
*. Breterm in<ants
to in<ants 5*
month
Am,icillin K .e<ota3ime
+. 1n<ants *%:
mos
Am,icillin K .e<ota3ime or
ce<tria3one
:. 1mm"nocom,e
tent chil)ren C:
mos an) a)"lts
5--
vancomycin K .e<ota3ime or
ce<tria3one
#. A)"lts C-- an)
a)"lts o< any a'e
with alcoholism or
other )ebilitatin'
illnesses
Am,icillin K ce<ota3ime or ce<tria3one
K vancomycin
-. Hos,italG
ac?"ire)
menin'itis@
,osttra"matic or
,ostne"ros"r'ery
menin'itis@
ne"tro,enic
,atients@ or
,atients with
im,aire) cellG
me)iate)
imm"nity
Am,icillin K ce<taHi)ime K
vancomycin
me
/ancomyc
in
0$ (m'(9')()@ ?0h + '()@ ?*+h
Specific 1ntimicrobial !herapy
>enin'ococcal >enin'itis
o Altho"'h ce<tria3one an) ce<ota3ime ,rovi)e a)e?"ate
em,irical covera'e <or N. meningitidis@ ,enicillin remains the
antibiotic o< choice <or menin'ococcal menin'itis ca"se) by
s"sce,tible strains
Bne"mococcal >enin'itis
o Antimicrobial thera,y o< ,ne"mococcal menin'itis is initiate)
with a ce,halos,orin (ce<tria3one@ ce<ota3ime@ or ce<e,ime)
an) vancomycin
o A +Gwee9 co"rse o< intraveno"s antimicrobial thera,y is
recommen)e) <or ,ne"mococcal menin'itis.
o Batients with S. pneumoniae menin'itis sho"l) have a re,eat
LB ,er<orme) +#%:0 h a<ter the initiation o< antimicrobial
thera,y to )oc"ment steriliHation o< the .S7
Listeria >enin'itis
o >enin'itis )"e to L. monocytogenes is treate) with am,icillin
<or at least : wee9s
o The combination o< trimetho,rim L*$%+$ (m'(9')()M an)
s"l<ametho3aHole L-$%*$$ (m'(9')()M 'iven every 0 h may
,rovi)e an alternative in ,enicillinGaller'ic ,atients.
Sta,hylococcal >enin'itis
o >enin'itis )"e to s"sce,tible strains o< S. aureus or coa'"laseG
ne'ative sta,hylococci is treate) with na<cillin
o /ancomycin is the )r"' o< choice <or methicillinGresistant
sta,hylococci an) <or ,atients aller'ic to ,enicillin.
o .S7 sho"l) be monitore) )"rin' thera,y. 1< the .S7 is not
steriliHe) a<ter #6 h o< intraveno"s vancomycin thera,y@ then
either intraventric"lar or intrathecal vancomycin@ +$ m' once
)aily@ can be a))e).
ramGNe'ative 4acillary >enin'itis
o The thir)G'eneration ce,halos,orinsNce<ota3ime@ ce<tria3one@
an) ce<taHi)imeNare e?"ally e<<icacio"s <or the treatment o<
'ramGne'ative bacillary menin'itis@ with the e3ce,tion o<
menin'itis )"e to P. aeruginosa@ which sho"l) be treate) with
ce<taHi)ime@ ce<e,ime@ or mero,enem
o A :Gwee9 co"rse o< intraveno"s antibiotic thera,y is
recommen)e) <or menin'itis )"e to 'ramGne'ative bacilli.
1*:U'C! !2+(1P/
;e3amethasone e3erts its bene<icial e<<ect by inhibitin' the
synthesis o< 1LG* an) TN7 at the level o< mRNA@ )ecreasin' .S7
o"t<low resistance@ an) stabiliHin' the bloo)Gbrain barrier.
The rationale <or 'ivin' )e3amethasone +$ min be<ore antibiotic
thera,y is that )e3amethasone inhibits the ,ro)"ction o< TN7 by
macro,ha'es an) micro'lia only i< it is a)ministere) be<ore these
cells are activate) by en)oto3in.
I'C(+1S+* I'!(1C(1'I1L P(+SSU(+
Emer'ency treatment o< increase) 1.B incl")es
*) elevation o< the ,atientFs hea) to :$%#-O@
2) int"bation an) hy,erventilation (Ba
.O+
+-%:$ mmH')@
:) mannitol.
10& P().')SIS
-)(!1LI!/
>ortality is :%&= <or menin'itis ca"se) by H. influenzae@ N.
meningitidis@ or 'ro", 4 stre,tococciE
*-= <or that )"e to L. monocytogenesE
+$= <or S. pneumoniae.
S+;U+L+
>o)erate or severe se?"elae occ"r in 8+-= o< s"rvivors@ altho"'h
the e3act inci)ence varies with the in<ectin' or'anism.
.ommon se?"elae incl")e
i. )ecrease) intellect"al <"nction@
ii. memory im,airment@
iii. seiH"res@
iv. hearin' loss an) )iHHiness@
v. 'ait )ist"rbances.
'eurocysticercosis
I'!()
Ne"rocysticercosis is the most common ,arasitic )isease o< the .NS
worl)wi)e.
H"mans ac?"ire cysticercosis by the in'estion o< <oo) contaminate) with
the e''s o< the ,arasite . solium .
Clinical Presentation
1. The most common mani<estation o< ne"rocysticercosis is newGonset
,artial seiH"res with or witho"t secon)ary 'eneraliHation.
2. Dhen ,resent in the s"barachnoi) or ventric"lar s,aces@ cysticerci can
,ro)"ce increase) 1.B
3. S,inal cysticerci can mimic the ,resentation o< intras,inal t"mors.
*iagnosis
The lesions o< ne"rocysticercosis are rea)ily vis"aliHe) by >R1 or .T
scans.
A very early si'n o< cyst )eath is hy,ointensity o< the vesic"lar <l"i) on
T+Gwei'hte) ima'es when com,are) with .S7.
Barenchymal brain calci<ications are the most common <in)in' an)
evi)ence that the ,arasite is no lon'er viable.
!reatment
Anticonv"lsant thera,y is initiate) when the ,atient with
ne"rocysticercosis ,resents with a seiH"re.
Antie,ile,tic thera,y can be sto,,e) once the <ollowG", .T scan shows
resol"tion o< the lesion.
Lon'Gterm antie,ile,tic thera,y is recommen)e) when
1)seiH"res occ"r a<ter resol"tion o< e)ema an) resor,tion
+) calci<ication o< the )e'eneratin' cyst.
.ystici)al )r"'s accelerate the )estr"ction o< the ,arasites@ res"ltin' in a
<aster resol"tion o< the in<ection.
alben)aHole *- m'(9' ,er )ay in two )oses <or 6 )ays.
,raHi?"antel -$ m'(9' ,er )ay <or *- )ays@
ACUTE STROKE
MANAGEMENT
*. BR1>AR! STEBS
1fter the clinical diagnosis of stro<e is made = an
or)erly ,rocess o< eval"ation an) treatment sho"l) <ollow
!he first goal is to ,revent or reverse brain inI"ry.
Atten) to the ,atientFs (15C)air>ay= breathing=
circulation@ an) treat hypoglycemia or hyperglycemia i<
i)enti<ie).
Ber<orm an emergency noncontrast head C! scan in
or)er to )i<<erentiate between ischemic stro9e an)
hemorrha'ic stro9eE there are no reliable clinical <in)in's
that concl"sively se,arate ischemia <rom hemorrha'e@
altho"'h
*. a more )e,resse) level o< conscio"sness@
+. hi'her initial bloo) ,ress"re@ or
:. worsenin' o< sym,toms a<ter onset
<avor hemorrha'e@ an) a )e<icit that remits s"''ests
ischemia.
+. >e)ical S",,ort
Dhen ischemic stro9e occ"rs@ the immediate goal is to
o,timiHe cerebral ,er<"sion in the s"rro"n)in' ischemic
,en"mbra.
1ttention is also directed to>ard preventing the
common complications of bedridden patientsN
in<ections (,ne"monia@ "rinary tract@ an) s9in) an) )ee,
veno"s thrombosis (;/T) with ,"lmonary embolism.
*. ,ne"matic com,ression stoc9in's to ,revent ;/TE
+. s"bc"taneo"s he,arin a,,ears to be sa<e as well an)
can be "se) concomitantly.
4eca"se collateral blood flo> >ithin the ischemic brain
is blood pressure dependent@ there is controversy abo"t
whether bloo) ,ress"re sho"l) be lowere) ac"tely.
5lood pressure should be lo>ered if
*. there is mali'nant hy,ertension
+. concomitant myocar)ial ischemia
:. i< bloo) ,ress"re is C*6-(**$ mmH' an) thrombolytic
thera,y is antici,ate).
?hen faced >ith the competing demands of
myocardium and brain@ lowerin' the heart rate with a P
*
G
a)rener'ic bloc9er (s"ch as esmolol) can be a <irst ste, to
)ecrease car)iac wor9 an) maintain bloo) ,ress"re.
Fever is )etrimental an) sho"l) be treate) with anti,yretics
an) s"r<ace coolin'.
Serum glucose sho"l) be monitore) an) 9e,t at 5**$
m'()L "sin' an ins"lin in<"sion.
4etween - TO *$= o< ,atients )evelo, enough cerebral
edema to cause obtundation or brain herniation.
E)ema ,ea9s on the secon) or thir) )ay b"t can ca"se mass
e<<ect <or 8*$ )ays. The lar'er the in<arct@ the 'reater the
li9elihoo) that clinically si'ni<icant e)ema will )evelo,.
?ater restriction and I@ mannitol may be used to
raise the serum osmolarity@ b"t hy,ovolemia sho"l) be
avoi)e) as this may contrib"te to hy,otension an)
worsenin' in<arction.
hemicraniectomy 7craniotomy and temporary removal
of part of the s<ull) mar9e)ly re)"ces mortality@ an) the
clinical o"tcomes o< s"rvivors are acce,table.
Prophylactic suboccipital decompression of large
cerebellar infarcts be<ore brainstem com,ression@ is
,ractice) at most stro9e centers
S,ecial vi'ilance is warrante) for patients >ith
cerebellar infarction.
S"ch stro9es may mimic labyrinthitis beca"se o< ,rominent
vertigo and vomitingE
the ,resence o< hea) or nec9 ,ain sho"l) alert the ,hysician
to consi)er cerebellar stro9e <rom vertebral artery
)issection.
:. 1ntraveno"s Thrombolysis
recombinant tP1 7rtP19
!he time of stro<e onset is )e<ine) as the
*. time the ,atientFs sym,toms be'an or
+. the time the ,atient was last seen as normal.
:. Batients who awa9en with stro9e have the onset
)e<ine) as when they went to be)
A)ministration o< 1ntraveno"s Recombinant Tiss"e Blasmino'en
Activator (rtBA) <or Ac"te 1schemic Stro9e
a
Indication
*. .linical )ia'nosis o<
stro9e
+. Onset o< sym,toms
to time o< )r"'
a)ministration 5A: h
:. .T scan showin' no
hemorrha'e or e)ema
o< CQ o< the >.A
territory
#. A'e CA*6 years
-. .onsent by ,atient
or s"rro'ate
Contraindication
*. S"staine) 4B C*6-(**$
)es,ite treatment
+. Blatelets 5*$$@$$$E
:. H.T 5+-=E
#. 'l"cose 5-$ or C#$$
m'()L
-. Use o< he,arin within #6
h an) ,rolon'e) BTT@ or
elevate) 1NR
0. Recent myocar)ial
in<arction
&. Brior stro9e or hea)
inI"ry within : monthsE
6. ,rior intracranial
hemorrha'e
2. >aIor s"r'ery in
,rece)in' *# )ays
*$. astrointestinal
blee)in' in ,rece)in' +*
)ays
**. >inor stro9e sym,toms
*+. Ra,i)ly im,rovin'
sym,toms
*:. .oma or st",or
1dministration of rtP1
*. 1ntraveno"s access with two ,eri,heral 1/ lines (avoi)
arterial or central line ,lacement)
+. Review eli'ibility <or rtBA
:. A)minister $.2 m'(9' intraveno"sly (ma3im"m 2$ m') as
*$= o< total )ose by bol"s@ <ollowe) by remain)er o< total
)ose over * h
#. 7re?"ent c"<< bloo) ,ress"re monitorin'
-. 7or )ecline in ne"rolo'ic stat"s or "ncontrolle) bloo)
,ress"re@ sto, in<"sion@ 'ive cryo,reci,itate@ an) reima'e
brain emer'ently
0. No other antithrombotic treatment <or +# h
&. Avoi) "rethral catheteriHation <or CA+ h
#. En)ovasc"lar Techni?"es
1schemic stro9e <rom largeAvessel intracranial occlusion
res"lts in hi'h rates o< mortality an) morbi)ity.
Occl"sions in s"ch lar'e vessels Lmi))le cerebral artery
(>.A)@ internal caroti) artery(1.A)@ an) the basilar
artery(4A)M generally involve a large clot volume an)
o<ten <ail to o,en with 1/ rtBA alone.
+ndovascular mechanical thrombectomy has recently
shown ,romise as an alternative treatment o< ac"te stro9e
in
*. ,atients who are ineli'ible <or@ or have
contrain)ications to@ thrombolytics or
+. in those who have <aile) to have vasc"lar
recanaliHation with 1/ thrombolytics.
en)ovasc"lar thrombectomy )evice to restore ,atency o<
occl")e) intracranial vessels >ithin " h of ischemic
stro<e symptoms&
-.Antithrombotic Treatment
Blatelet 1nhibition
As,irin is the only anti,latelet a'ent that has been ,roven
e<<ective <or the ac"te treatment o< ischemic stro9eE
0.Ne"ro,rotection
Ne"ro,rotection is the conce,t o< ,rovi)in' a treatment that
,rolon's the brainFs tolerance to ischemia.
Hy,othermia is a ,ower<"l ne"ro,rotective treatment in
,atients with car)iac arrest an) is ne"ro,rotective in animal
mo)els o< stro9e@ b"t it has not been a)e?"ately st")ie) in
,atients with ischemic stro9e.
&.Stro9e .enters an) Rehabilitation
Batient care in com,rehensive stro9e "nits <ollowe) by
rehabilitation services improves neurologic outcomes
and reduces mortality&
.
Proper rehabilitation of the stro<e patient includes
early ,hysical@ occ",ational@ an) s,eech thera,y.
1t is )irecte) towar)
*. e)"catin' the ,atient an) <amily abo"t the ,atientFs
ne"rolo'ic )e<icit@
+. ,reventin' the com,lications o< immobility
:. ,rovi)in' enco"ra'ement an) instr"ction in
overcomin' the )e<icit.
!he goal of rehabilitation is
*. to ret"rn the ,atient to home an)
+. to ma3imiHe recovery by ,rovi)in' a sa<e@ ,ro'ressive
re'imen s"ite) to the in)ivi)"al ,atient.

6.Restraint thera,y (immobiliHin' the


"na<<ecte) si)e) has been shown to im,rove
hemi,aresis <ollowin' stro9e@ even years <ollowin' the stro9e@
s"''estin' that ,hysical thera,y can recr"it "n"se) ne"ral
,athways.
This <in)in' s"''ests that the h"man nervo"s system is
more a)a,table than ori'inally tho"'ht an) has stim"late)
active research into ,hysical an) ,harmacolo'ic strate'ies
that can enhance lon'Gterm ne"ral recovery
C+(+5(1L @+')US
!2()-5)SIS A -
B
-edical Care
>e)ical mana'ement o< the ,atient with cerebral veno"s thrombosis
(./T) is similar to that o< ,atients with arterial stro9e as <ar as stabiliHin'
the ,atient is concerne).
Batients with altere) mental stat"s or hemi,le'ia sho"l) be 'iven
nothin' by mo"th to ,revent as,iration.
1ntraveno"s <l"i)s sho"l) not be hy,otonic sol"tions. Normal saline is
recommen)e) at a rate o< a,,ro3imately *$$$ mL in +# ho"rs.
To )ecrease intracranial ,ress"re@ the hea) sho"l) be elevate) :$G#$O at
all times.
s",,lemental o3y'en only when level o< conscio"sness is )ecrease).
SeiH"res sho"l) be treate) with a,,ro,riate anticonv"lsants.
7os,henytoin is recommen)e) <or treatment o< seiH"res in those
,atients who re?"ire a ,arenteral <orm"lation.
Alternatively@ ,henobarbital or so)i"m val,roate inIection may be
"tiliHe) i< the ,atient has aller'y to ,henytoin.
;iaHe,am or loraHe,am may be "se) to treat stat"s e,ile,tic"s@ b"t the
,atient also sho"l) be 'iven an anticonv"lsant with a lon'er )"ration o<
action to ,revent rec"rrent seiH"res.
Specific therapy for C@!
1nti coagulation
sho"l) be consi)ere) serio"sly in the mana'ement o< cerebral veno"s
thrombosis (./T).
.onversion to war<arin as maintenance thera,y is then s"''este).
,revent ,ro,a'ation o< the clot to more e3tensive areas o< the cerebral
veno"s system.
2eparin
1dult
1nitial in<"sionR *6 U(9'(h 1/E aBTT chec9e) in 0 h an) ?0h a<ter any )osa'e
chan'e@ as well as every amE
a)I"st )ose accor)in' to <ollowin' ,arameters
aBTT A 5*.+ times controlR 6$ U(9' bol"s with increase o< # U(9'(h
aBTT A *.+G*.- times controlR #$ U(9' bol"s with increase o< + U(9'(h
aBTT A *.-G+.: times controlR No chan'e in in<"sion rate nee)e)
aBTT A +.:G: times controlR ;ecrease in<"sion rate by + U(9'(h
aBTT C : times controlR Hol) in<"sion <or * h an) )ecrease rate by :
U(9'(h
?arfarin
war<arin treatment sho"l) be maintaine) <or :G0 mo
1dult
1nitialR - m' BO ?)E a)I"st )ose by monitorin' 1NR (tar'et@ +.-)
!hrombolytics
These a'ents ca"se lysis o< the clot.
This treatment at ,resent is limite) to s,ecialiHe) centers b"t sho"l) be
consi)ere) <or ,atients with si'ni<icant )e<icit.
Uro<inase
iven in ./T by catheteriHation o< veno"s sin"s or by )irect instillation at
s"r'ery )"rin' thrombectomy.
1dult
+@-$@$$$ U(h instille) )irectly or via veno"s sin"s catheterE a))itional )oses
o< -$@$$$ UE total )ose *@$$$@$$$ U over + h
Strepto<inase
1dult
1nstille) )irectly or via veno"s sin"s catheter
Surgical Care
1n cases o< severe ne"rolo'ic )eterioration@ o,en thrombectomy an)
local thrombolytic thera,y have been )escribe) as bene<icial.
Recently@ )ecom,ressive craniectomy has been re,orte) as a treatment
strate'y@ with varie) res"lts.

Lateral medullary syndrome
*+FI'I!)'
Lateral me)"llary syn)rome (also calle) Dallenber'
syn)rome an) ,osterior in<erior cerebellar artery
syn)romeSB1.AT) is a )isease in which the ,atient has a
constellation o< ne"rolo'ic sym,toms )"e to inI"ry to the
lateral ,art o< the me)"lla @ res"ltin' in tiss"e ischemia an)
necrosis.
C1US+S
occlusion of any of five vessels may be responsibleN
*) vertebral@
+) ,osterior in<erior cerebellar@
:) s",erior lateral me)"llary arteries@
#) mi))le lateral me)"llary arteries@
-) in<erior lateral me)"llary arteries
CLI'IC1L F+1!U(+S
!his syndrome is characteriCed by
*. sensory )e<icits a<<ectin' the tr"n9 an) e3tremities on the
o,,osite si)e o< the in<arction
+. sensory )e<icits a<<ectin' the <ace an) cranial nerves on
the same si)e with the in<arct.
!his crossed finding is )ia'nostic <or the syn)rome.
)n side of lesion
*) Bain@ n"mbness@ im,aire) sensation over hal< the <aceR
Descending tract and nucleus fift ner!e
+) Ata3ia o< limbs@ <allin' to si)e o< lesionR !ncertain"restif"r#
$"d%& cere$ellar e#is'ere& cere$ellar fi$ers&
s'in"cere$ellar tract ()*
:) Nysta'm"s@ )i,lo,ia@ oscillo,sia@ verti'o@ na"sea@ vomitin'R
+esti$ular nucleus
#) HornerFs syn)rome (miosis@ ,tosis@ )ecrease) sweatin')R
Descending s%#'atetic tract
-) ;ys,ha'ia@ hoarseness@ ,aralysis o< ,alate@ ,aralysis o< vocal
cor)@ )iminishe) 'a' re<le3R Issuing fi$ers nint and tent
ner!es
0) Loss o< tasteR Nucleus and tractus s"litaries
&) N"mbness o< i,silateral arm@ tr"n9@ or le'R Cuneate and gracile
nuclei
6) Dea9ness o< lower <aceR Genuflected u''er #"t"r
neur"n(UMN* fi$ers t" i'silateral facial nucleus
)n side opposite lesion
*) 1m,aire) ,ain an) thermal sense over hal< the bo)y@ sometimes
<aceR S'in"tala#ic tract
!(+1!-+'!
Treatment <or lateral me)"llary syn)rome involves <oc"sin'
on
*. relie< o< sym,toms an)
+. active rehabilitation
1>>E;1ATE
1. A <ee)in' t"be inserte) thro"'h the mo"th or 'astrostomy
may be necessary if s>allo>ing is impaired.
2. S,eech thera,y may be bene<icial
3. 1n some cases@ me)ication may be "se) to re)"ce or
eliminate pain .
4. One o< the most "ni?"e an) )i<<ic"lt to treat sym,toms that
occ"r )"e to Dallenber' syn)rome are interminable=
violent hiccups. The hicc",s can be so severe that ,atients
o<ten str"''le to eat@ slee, an) carry on conversations.
5. ;e,en)in' on the severity o< the bloc9a'e ca"se) by the
stro9e@ the hicc",s can last <or wee9s. Un<ort"nately there
are very <ew s"ccess<"l me)ications available to me)iate the
inconvenience o< constant hicc",s.
LON TER>
1. Long term treatment 'enerally involves the "se o< bloo)
thinners li9e war<arin.
2. Batients will o<ten remain on these me)ications or an as,irin
re'imen <or the rest o< their lives in or)er to minimiHe the
ris9 o< another stro9e.
3. Other me)ications may be necessary in or)er to s",,ress
hi'h bloo) ,ress"re an) ris9 <actors associate) with stro9es.
!ransient Ischemic
1ttac<s
*+FI'I!I)'
T1As are e,iso)es o< stro9e sym,toms that last only brie<lyE the
stan)ar) )e<inition o< )"ration is 5+# h@ b"t most T1As last 5* h.
#maurosis fuga$@ or transient monoc"lar blin)ness@ occ"rs <rom
emboli to the central retinal artery o< one eye. This may in)icate
caroti) stenosis as the ca"se or local o,hthalmic artery )isease.
(ISD
The ris9 o< stro9e a<ter a T1A is 8*$%*-= in the <irst : months@
with most events occ"rrin' in the <irst + )ays.
(is< Factors for Ischemic Stro<e and !I1
1)enti<ication an) control o< mo)i<iable ris9 <actors is the best
strate'y to re)"ce the b"r)en o< stro9e@ an) the total n"mber o<
stro9es co"l) be re)"ce) s"bstantially by these means
*. Hy,ertension
+. Atrial <ibrillation
:. ;iabetes
#. Smo9in'
-. Hy,erli,i)emia
0. Asym,tomatic caroti)
stenosis
&. Sym,tomatic caroti) stenosis
(&$%22=)
6. Sym,tomatic caroti) stenosis
(-$%02=)
C1US+S
Common Causes
Uncommon Causes
Thrombosis
Lac"nar stro9e (small
vessel)
Lar'e vessel thrombosis
;ehy)ration
Embolic occl"sion
ArteryGtoGartery
.aroti) bi<"rcation
Aortic arch
Hy,ercoa'"lable )isor)ers
/eno"s sin"s thrombosis
7ibrom"sc"lar )ys,lasia

/asc"litis
.ar)io'enic
Uncommon Causes
Arterial )issection
.ar)ioembolic
Atrial <ibrillation
>"ral thromb"s
>yocar)ial in<arction
;ilate) car)iomyo,athy
/alv"lar lesions
>itral stenosis
>echanical valve
4acterial en)ocar)itis
Bara)o3ical embol"s
Atrial se,tal )e<ect
Batent <oramen ovale
>itral valve calci<ication
Atrial my3oma
1ntracar)iac t"mor
S"barachnoi) hemorrha'e
vasos,asm

;r"'sR cocaine@ am,hetamine

Eclam,sia
-1'1.+-+'! PL1'
71 :0#G* HARR1SON
Primary and Secondary Prevention of Stro<e and !I1
eneral Brinci,les
A n"mber o< me)ical an) s"r'ical interventions@ as well as
li<estyle mo)i<ications@ are available <or ,reventin' stro9e.
Atherosclerosis Ris9 7actors
statin )r"'s re)"ce the ris9 o< stro9e
,atients with recent stro9e or T1A G ,rescribe atorvastatin@ 6$
m'().
Tobacco smo9in' sho"l) be )isco"ra'e) in all ,atients
Anti,latelet A'ents
Platelet antiaggregation agents can ,revent atherothrombotic
events@ incl")in' T1A an) stro9e
As,irin@ clo,i)o'rel@ an) the combination o< as,irin ,l"s e3ten)e)G
release )i,yri)amole are the anti,latelet a'ents most commonly
"se) <or this ,"r,ose.
Ticlo,i)ine has been lar'ely aban)one) beca"se o< its a)verse
e<<ects.
The choice o< anti,latelet a'ent an) )ose m"st balance the ris9 o<
stro9e@ the e3,ecte) bene<it@ an) the ris9 an) cost o< treatment.
Anticoa'"lation Thera,y an) Embolic Stro9e
Several trials have shown that anticoa'"lation (1NR ran'e@ +%:) in
,atients with chronic nonvalv"lar (nonrhe"matic) atrial <ibrillation
,revents cerebral embolism an) is sa<e.
7or ,rimary ,revention an) <or ,atients who have e3,erience)
stro9e or T1A@ anticoa'"lation with war<arin re)"ces the ris9 by
abo"t 0&=@ which clearly o"twei'hs the *= ris9 ,er year o< a
maIor blee)in' com,lication.
1< the embolic so"rce cannot be eliminate)@ anticoa'"lation sho"l)
in most cases be contin"e) in)e<initely.
>any ne"rolo'ists recommen) combinin' anti,latelet a'ents with
anticoa'"lants <or ,atients who U<ailU anticoa'"lation (i.e.@ have
another stro9e or T1A).
?eberEs syndrome
I'!()
?eberEs syndrome is a <orm o< stro9e characteriHe) by the ,resence o<
an oc"lomotor nerve ,alsy an) contralateral hemi,aresis or hemi,le'ia.
Cause
o 1t is ca"se) by mi)brain in<arction as a res"lt o< occl"sion o< the
,arame)ian branches o< the ,osterior cerebral artery (B.A) or o<
basilar bi<"rcation ,er<oratin' arteries.
o DeberFs Syn)rome has ,resente) as a mani<estation o<
)ecom,ression illness in a recreational sc"ba )iver.
L+M
Presentation
This lesion is "s"ally "nilateral an) a<<ects several str"ct"res in the mi)brain
incl")in'R
Structure
damaged
+ffect
*. s"bstantia
ni'ra
contralateral ,ar9insonism
+. corticos,inal
<ibers
contralateral hemi,aresis an) ty,ical ",,er motor
ne"ron <in)in's
:. corticob"lbar
tract
)i<<ic"lty with contralateral lower <acial m"scles an)
hy,o'lossal nerve <"nctions
#. oc"lomotor i,silateral oc"lomotor nerve ,alsy with a )roo,in'
nerve <ibers
eyeli) an) <i3e) wi)e ,",il ,ointe) )own an) o"t. This
lea)s to )i,lo,ia
5enedi<t syndrome
I'!()
5enedi<t syndrome@or paramedian midbrain syndrome@ is a rare
ty,e o< ,osterior circ"lation stro9e o< the brain@ with a ran'e o<
ne"rolo'ical sym,toms a<<ectin' the mi)brain@ cerebell"m an) other
relate) str"ct"res
Causes
4ene)i9t syn)rome is ca"se) by a lesion ( in<arction@ hemorrha'e@
t"mor@ or t"berc"losis) in the te'ment"m o< the mi)brain an)
cerebell"m.
S,eci<ically@ the me)ian Hone is im,aire).
1t can res"lt <rom occl"sion o< the ,osterior cerebral artery.
CharacteriCation
'euroanatomical str"ct"res a<<ecte)
incl")e .N111 n"cle"s@
Re) n"cle"s@
corticos,inal tracts@
brachi"m conI"nctiv"m@ an)
cerebell"m.
1t is characteriHe) by the ,resence o<
1)an .N 111 oc"lomotor nerve ,alsy an)
2)contralateral hemi,aresis (wea9ness) an)
3)cerebellar ata3ia incl")in' tremor.
!reatment
;ee, brain stim"lation may ,rovi)e relie< <rom some sym,toms o<
4ene)i9t syn)rome@ ,artic"larly the tremors associate) with the )isor)er
FovilleEs syndrome
I'!()
FovilleEs syndrome is ca"se) by the bloc9a'e o< the ,er<oratin'
branches o< the basilar artery in the ,ons.
Structures affected
1. the BBR7@ (,arame)ian ,ontine retic"lar <ormation)
2. n"clei o< cranial nerves /1 an) /11@
3. corticos,inal tract@
4. me)ial lemnisc"s@
5. the me)ial lon'it")inal <ascic"l"s.
Presentation
1. i,silateral horiHontal 'aHe ,alsy
2. <acial nerve ,alsy
3. contralateral hemi,aresis@
#. hemisensory loss@
5. intern"clear o,hthalmo,le'ia.
-UL!IPL+
SCL+()SIS
;E71N1T1ON
;emyelinatin' )isor)ers characteriHe) by in<lammation an)
selective )estr"ction o< central nervo"s system (.NS) myelin. The
,eri,heral nervo"s system (BNS) is s,are)@ an) most ,atients
have no evi)ence o< an associate) systemic illness.
.HARA.TER1ST1. 7EATURES
>"lti,le sclerosis (>S) is characteriHe) by a tria) o< in<lammation@
)emyelination@ an) 'liosis (scarrin')E
the co"rse can be rela,sin'Gremittin' or ,ro'ressive.
Lesions o< >S ty,ically occ"r at )i<<erent times an) in )i<<erent
.NS locations (i.e.@ )isseminate) in time an) s,ace).
EB1;E>1OLO!
>S is a,,ro3imately three<ol) more common in women than men.
The a'e o< onset is ty,ically between +$ an) #$ years@ b"t the
)isease can ,resent across the li<es,an.
eo'ra,hical 'ra)ients have been re,eate)ly observe) in >S@
with ,revalence rates increasin' at hi'her latit")es. hi'h rates are
<o"n) thro"'ho"t northern E"ro,e@ the northern Unite) States@
an) .ana)a. 4y contrast@ the ,revalence is low in Va,an @ in other
,arts o< Asia@ in e?"atorial A<rica@ an) in the >i))le East.
viral in<ections (e.'.@ ,oliomyelitis an) measles vir"ses)@ h"man
her,es vir"s ty,e 0 (HH/G0) or %hlamydia pneumonia, remote
E,steinG4arr vir"s (E4/) in<ection
Evi)ence also s",,orts an im,ortant 'enetic in<l"ence on >S
.linical >ani<estations
The onset o< >S may be abr",t or insi)io"s.
Clinical course of multiple sclerosis 7-S9&
A, Rela,sin'(remittin' >S. (RR)
B, Secon)ary ,ro'ressive >S. (SB)
C, Brimary ,ro'ressive >S. (BB)
D, Bro'ressive(rela,sin' >S.(BR)
Sym,toms may be severe or seem so trivial that a ,atient may
not see9 me)ical attention <or months or years.
1nitial Sym,toms o< >S
Symptom
*. Sensory loss
+. O,tic ne"ritis
:. Dea9ness
#. Baresthesias
-. ;i,lo,ia
0. Ata3ia
&. /erti'o
6. Baro3ysmal attac9s
2. Lhermitte
*$. Bain
**. ;ementia
*+. /is"al loss
Symptom
*:. 7acial ,alsy
*#. 1m,otence
*-. >yo9ymia
*0. E,ile,sy
*&. 7allin'
E3erciseGin)"ce) wea9ness is a characteristic sym,tom o< >S
Heat sensiti&ity re<ers to ne"rolo'ic sym,toms ,ro)"ce) by an
elevation o< the bo)yFs core tem,erat"re. 7or e3am,le@ "nilateral
vis"al bl"rrin' may occ"r )"rin' a hot shower or with ,hysical
e3ercise (!hthoff's symptom)
Lhermitte's symptom is an electric shoc9 li9e sensation (ty,ically
in)"ce) by <le3ion or other movements o< the nec9) that ra)iates
)own the bac9 into the le's. Rarely@ it ra)iates into the arms. 1t is
'enerally sel<Glimite) b"t may ,ersist <or years. LhermitteFs
sym,tom can also occ"r with other )isor)ers o< the cervical s,inal
cor) (e.'.@ cervical s,on)ylosis).
;ia'nostic .riteria
;ia'nostic .riteria <or >S
*. E3amination m"st reveal ob(ecti&e abnormalities o< the .NS.
+. 1nvolvement m"st re<lect ,re)ominantly )isease o< white
matter lon' tracts@ "s"ally incl")in'
(a) ,yrami)al ,athways@
(b) cerebellar ,athways@
(c) me)ial lon'it")inal <ascic"l"s@
()) o,tic nerve@
(e) ,osterior col"mns.
:. E3amination or history m"st im,licate involvement o< two or
more areas o< the .NS.
(a). >R1 may be "se) to )oc"ment a secon) lesion when only one
site o< abnormality has been )emonstrable on e3amination.
A con<irmatory >R1 m"st have either <o"r lesions involvin' the
white matter or three lesions i< one is ,eriventric"lar in
location.
Acce,table lesions m"st be C: mm in )iameter.
(b). Evo9e) res,onse testin' may be "se) to )oc"ment a secon)
lesion not evi)ent on clinical e3amination.
#. The clinical ,attern m"st consist o<
(a) two or more se,arate e,iso)es o< worsenin' involvin'
)i<<erent sites o< the .NS@ each lastin' at least +# h an) occ"rrin'
at least * month a,art@ or
(b) 'ra)"al or ste,wise ,ro'ression over at least 0 months i<
accom,anie) by increase) 1' synthesis or two or more
oli'oclonal ban)s.
-. The ,atientFs ne"rolo'ic con)ition co"l) not better be
attrib"te) to another )isease.
*iagnostic Categories
*. )efinite MS* All <ive criteria <"l<ille).
+. Probable MS* All <ive criteria <"l<ille) e3ce,t
(a) only one obIective abnormality )es,ite two sym,tomatic
e,iso)es or
(b) only one sym,tomatic e,iso)e )es,ite two or more obIective
abnormalities.
:. #t ris+ for MS* .riteria *@ +@ :@ an) - <"l<ille)E ,atient has only
one sym,tomatic e,iso)e an) one obIective abnormality.
;177ERENT1AL ;1ANOS1S
;isor)ers that .an >imic >S
*. Ac"te )isseminate) ence,halo myelitis (A;E>)
+. Anti,hos,holi,i) antibo)y syn)rome
:. 4ehJetFs )isease
#. H"man imm"no)e<iciency vir"s (H1/) in<ection
-. Lyme )isease
0. Neo,lasms (e.'.@ lym,homa@ 'lioma@ menin'ioma)
&. Sarcoi)
6. SIW'renFs syn)rome
2. Stro9e an) ischemic cerebrovasc"lar )isease
*$. Sy,hilis
**. Systemic l","s erythematos"s an) relate) colla'en vasc"lar
)isor)ers
*+./itamin 4
*+
)e<iciency
1N/EST1AT1ONS RE7ER ;A/1;SON
>ANAE>ENT RE7ER ;A/1;SON
-igraine 2eadache
INTRO - TRIGGERS
>i'raine@ the secon) most common ca"se o< ,rimary
hea)ache@
a<<ects a,,ro3imately *-= o< women an) 0= o< men.

Sym,toms Accom,anyin' Severe >i'raine Attac9s
Symptom
*) Na"sea
+) /omitin'
:) ;iarrhea
#) Li'ht hea)e)ness
-) Scal, ten)erness
0) /is"al )ist"rbances
&) Bhoto,hobia
6) Bhoto,sia
2) /erti'o
*$) SeiH"re
**) Synco,e
*+) Alteration o< conscio"sness
*:) .on<"sional state
*#) Baresthesias

Hea)ache can be initiate) or am,li<ie) by vario"s tri''ers@
incl")in'
*) 'lare@ bri'ht li'hts@
+) so"n)s@
:) h"n'erE
#) e3cess stressE
-) ,hysical e3ertionE
0) stormy weather or
&) barometric ,ress"re chan'esE
6) hormonal <l"ct"ations )"rin' mensesE
2) lac9 o< or e3cess slee,E an)
*$) alcohol or other chemical stim"lation.
Dno>ledge of a patientEs susceptibility to s,eci<ic
tri''ers can be "se<"l in mana'ement strate'ies involvin'
li<estyle a)I"stments.
Diagn"sis and Clinical Features
1 high indeF of suspicion is re?"ire) to )ia'nose
mi'raineR
migraine aura@ consistin' o< vis"al )ist"rbances with
<lashin' li'hts or Hi'Ha' lines movin' across the vis"al <iel)
or o< other ne"rolo'ic sym,toms@ is re,orte) in only +$%+-=
o< ,atients
Patients >ith episodes of migraine that occur daily or
nearAdaily are consi)ere) to have chronic mi'raine .
-igraine must be differentiated from tensionAtype
headache @ the most common ,rimary hea)ache syn)rome
seen in clinical ,ractice.
Migraine at its most basic le&el is eadace .it
ass"ciated features, and tension,type headache is
headache that is featureless. Most patients -ith disabling
headache probably ha&e migraine.
Sim,li<ie) ;ia'nostic .riteria <or >i'raine
Re,eate) attac9s o< hea)ache lastin' #%&+ h in ,atients
with a normal ,hysical e3amination@
no other reasonable ca"se <or the hea)ache@
an)R
1t least 2 of the follo>ing
featuresG
Plus at least 1 of the
follo>ing featuresG
*. Unilateral ,ain *. Na"sea(vomitin'
+. Throbbin' ,ain +. Bhoto,hobia an)
,hono,hobia
:. A''ravation by
movement

#. >o)erate or severe
intensity

Batients with ace,hal'ic mi'raine e3,erience rec"rrent
ne"rolo'ic sym,toms@ o<ten with na"sea or vomitin'@ b"t
with little or no hea)ache.
/erti'o can be ,rominentE it has been estimate) that oneG
thir) o< ,atients re<erre) <or verti'o or )iHHiness have a
,rimary )ia'nosis o< mi'raine.
MANAGEMENTG
re<er K;T
CLUS!+( 2+1*1C2+
Cluster 2eadache
*. .ender >C7
+. Pain
Ty,e Stabbin'@ borin'
Severity E3cr"ciatin'
Site Orbit@ tem,le
:. 1ttac<
freHuency
*(alternate )ay% 6()
#. *uration of
attac<
*-%*6$ min
-. 1utonomic
features
!es
3& -igrainous
features
!es
&. 1lcohol
trigger
!es
Cluster 2eadache
6. Cutaneous
triggers
No
2. 1bortive
treatment
S"matri,tan inIection or
nasal s,ray
O3y'en
*$.Prophylactic
treatment
/era,amil
>ethyser'i)e
Lithi"m
!oFic
headache
To3ic hea)ache 1s "s"ally ca"se) by <ever <rom ac"te bacterial illnesses
or <rom e3,os"re to vario"s chemicals incl")in' <rom <"mes@ ,oll"tion
an) aller'ens.
Causes
*) 'itrite.
+) carbon tetrachloride
:) organophosphate pesticides=
#) acetal)ehy)e <rom alcohol 7a hangover)
-) !oFic shoc< syndrome
0) carbon monoFide poisoning.
&) amphetamines
!reatment
.a<<eine can be "se) to alleviate a vasc"lar hea)ache by constrictin'
)ilate) arteries.
C! vs -(I
C! Scans
INDICATIONS
1< initial eval"ations in)icate a s9"ll <ract"re or intracranial
blee)in'@ a .T (or .AT) scan is "s"ally or)ere).
The .T scans can reveal
*) hematomas@
+) hemorrha'es@ an)
:) s9"ll <ract"res
'ivin' the ne"rolo'ist e3actly the in<ormation necessary <or
)eci)in' i< emer'ency treatment is nee)e) an) ,recisely
where.
TEC/NI0UE
a way to obtain )etaile) XGray ,ict"res o< crossGsections
thro"'h the bo)y.
AD+ANTAGE
Testin' is <ast an) res"lts are ?"ic9E ma9in' it e3ce,tionally
val"able when ,rom,t )ia'nosis an) treatment are critical.
Unli9e some other scannin' metho)s@ the .T scan can be
ta9en while the ,atient is hoo9e) ", to 1/Ys or other me)ical
e?"i,ment.
DRAA1BACKS
1rra)iation
-(I
INDICATIONS
>a'netic Resonance 1ma'in' (Z>R1Z) is not o<ten "se) in
ac"te hea) inI"ry cases.
A<ter the ac"te ,hase has ,asse)@ the )octor may want an
>R1 to eval"ate the location an) e3tent o< brain inI"ry to
)etermine <"rther treatment an) rehabilitation o,tions.
TEC/NI0UE
>R1 "ses ,ower<"l ma'netic <iel)s an) the ma'netic
reaction o< the bo)yYs cells to constr"ct crossGsectional
ima'es Similar to .T scans.
AD+ANTAGE
*) 4eca"se it )oesnYt "se XGrays@ it can be sa<er than .T i<
m"lti,le ima'in' sessions are e3,ecte).
+) /ariations o< >R1 technolo'y can also e3amine brain
<"nctionin' an) i)enti<y inI"ries not visible in .T scans. 4"t
even the )etail available "sin' >R1 cannot )etect mil)
conc"ssions.
:) 1t 'ives <iner )etails than .T.
DRA1BACKS INCLUDE2
*) Lon'er to ,er<orm
+) Not as rea)ily available as a .T scanner in most hos,itals
:) 1s not ,ractical <or ,atients hoo9e) ", to me)ical
e?"i,ment
#) .annot be "se) i< ,atient has metal embe))e) anywhere
in the bo)y
-) 1s not tolerate) well by some ,atients beca"se o< the
con<ine) s,ace insi)e the >R1 machine
0) E3,ensive
1lCheimerEs
*isease
1NTRO
A; can occ"r in any )eca)e o< a)"lthoo)@ b"t it is the most common
ca"se o< )ementia in the el)erly.
.linical >ani<estations
The co'nitive chan'es with A; ten) to <ollow a characteristic ,attern@
be'innin' with memory im,airment an) s,rea)in' to lan'"a'e an)
vis"os,atial )e<icits.
1n the early sta'es o< the )isease@
*) the memory loss may 'o "nreco'niHe) or be ascribe) to
beni'n <or'et<"lness.
2) Once the memory loss be'ins to a<<ect )ayGtoG)ay activities the
)isease is )e<ine) as >.1. A,,ro3imately -$= o< >.1 in)ivi)"als
will ,ro'ress to A; within - years.
:) Slowly the co'nitive ,roblems be'in to inter<ere with )aily
activities@ s"ch as 9ee,in' trac9 o< <inances@ <ollowin' instr"ctions
on the Iob@ )rivin'@ sho,,in'@ an) ho"se9ee,in'.
1n the mi))le sta'es o< A;@
*) the ,atient is "nable to wor9@ is easily lost an) con<"se)@ an)
re?"ires )aily s",ervision.
+) Social 'races@ ro"tine behavior@ an) s",er<icial conversation may
be s"r,risin'ly intact.
3) Lan'"a'e becomes im,aire)N<irst namin'@ then com,rehension@ an)
<inally <l"ency. 1n some ,atients@ aphasia is an early an) ,rominent
<eat"re.
4) #pra$ia emer'es@ an) ,atients have tro"ble ,er<ormin' se?"ential
motor tas9s.
-) /is"os,atial )e<icits be'in to inter<ere with )ressin'@ eatin'@
solvin' sim,le ,"HHles@ an) co,yin' 'eometric <i'"res. Batients may
be "nable to )o sim,le calc"lations or tell time.
1n the late sta'es o< the )isease@
*) some ,ersons remain amb"latory b"t wan)er aimlessly.
+) Loss o< I")'ment@ reason@ an) co'nitive abilities is inevitable.
3) ;el"sions are common an) "s"ally sim,le in ?"ality@ s"ch as
)el"sions o< the<t@ in<i)elity@ or misi)enti<ication.
4) A,,ro3imately *$= o< A; ,atients )evelo, %apgras' syndrome @
believin' that a care'iver has been re,lace) by an im,ostor.
5) Some ,atients )evelo, a sh"<<lin' 'ait with 'eneraliHe) m"scle
ri'i)ity associate) with slowness an) aw9war)ness o< movement.
1n en)Gsta'e A;@
*) ,atients become ri'i)@ m"te@ incontinent@ an) be)ri))en.
+) Hel, may be nee)e) with the sim,lest tas9s@ s"ch as eatin'@
)ressin'@ an) toilet <"nction.
3) They may show hy,eractive ten)on re<le3es.
4) >yoclonic Ier9s may occ"r s,ontaneo"sly or in res,onse to ,hysical
or a")itory stim"lation.
5) eneraliHe) seiH"res may also occ"r.
)eath res"lts <rom
*) maln"trition@
+) secon)ary in<ections@
3) ,"lmonary emboli@
#) heart )isease.
The ty,ical )"ration o< A; is 6%*$ years@ b"t the co"rse can ran'e <rom *
to +- years.
;1ANOS1S
Early in the )isease co"rse@ other etiolo'ies o< )ementia sho"l) be
e3cl")e).
Ne"roima'in' st")ies (.T an) >R1) )o not show a sin'le s,eci<ic ,attern
with A; an) may be normal early in the co"rse o< the )isease.
As A; ,ro'resses@ )i<<"se cortical atro,hy becomes a,,arent@ an) >R1
scans show atro,hy o< the hi,,ocam,"s .
1ma'in' hel,s to e3cl")e other )isor)ers@ s"ch as ,rimary an) secon)ary
neo,lasms@ m"lti in<arct )ementia@ )i<<"se white matter )isease@ an)
NBHE
it also hel,s to )istin'"ish A; <rom other )e'enerative )isor)ers with
)istinctive ima'in' ,atterns s"ch as 7T; or .V;.
7"nctional ima'in' st")ies in A; reveal hy,o,er<"sion or
hy,ometabolism in the ,osterior tem,oralG,arietal corte3
The EE in A; is normal or shows nons,eci<ic slowin'.
Ro"tine s,inal <l"i) e3amination is also normal.
Treatment
The mana'ement o< A; is challen'in' an) 'rati<yin'@
The ,rimary <oc"s is on lon'Gterm amelioration o< associate) behavioral
an) ne"rolo'ic ,roblems.
4"il)in' ra,,ort with the ,atient@ <amily members@ an) other care'ivers
is essential to s"ccess<"l mana'ement.
1n the early sta'es o< A;@ memory ai)s s"ch as noteboo9s an) ,oste)
)aily remin)ers can be hel,<"l.
;one,eHil@ rivasti'mine@ 'alantamine@ memantine@ an) tacrine are the
)r"'s ,resently a,,rove) by the 7oo) an) ;r"' A)ministration (7;A) <or
treatment o< A;. ;"e to he,atoto3icity@ tacrine is no lon'er "se).
1n ,atients with mo)erately a)vance) A;@ a ,ros,ective trial o< the
antio3i)ants sele'iline @ toco,herol (vitamin E)@ or both@ slowe)
instit"tionaliHation an) ,ro'ression to )eath.
4eca"se vitamin E has less ,otential <or to3icity than sele'iline an) is
chea,er@ the )oses "se) in this st")y o< *$$$ 1U twice )aily are o<<ere)
to many ,atients with A;.

>il) to mo)erate )e,ression is common in the early sta'es o< A; an)
res,on)s to anti)e,ressants or cholinesterase inhibitors.
Selective serotonin re",ta9e inhibitors (SSR1s) are commonly "se) )"e
to their low anticholiner'ic si)e e<<ects.
eneraliHe) seiH"res sho"l) be treate) with an a,,ro,riate
anticonv"lsant@ s"ch as ,henytoin or carbamaHe,ine.
The newer 'eneration o< aty,ical anti,sychotics@ s"ch as ris,eri)one an)
olanHa,ine@ are bein' "se) in low )oses to treat these ne"ro,sychiatric
sym,toms.
+B!(1 P/(1-I*1L
S/S!+-
1NTRO
The e$trapyramidal system is com,ose) o< motor <ibers which )o not
,ass thro"'h the me)"llary ,yrami)s b"t which nevertheless e3ert a
meas"re o< control over bo)ily movements.
4AS1. BARTS
the e3tra,yrami)al system can be )ivi)e) into three controllin' systemsR
*) cortically ori'inatin' in)irect ,athways@
+) <ee)bac9 loo,s@
:) a")itoryGvis"alGvestib"lar )escen)in' ,athways.
.ortically Ori'inatin' 1n)irect ;escen)in' Bathways
o At the same time si'nals are bein' transmitte) over the ,yrami)al
system to ,ro)"ce a s,eci<ic movement@ a))itional si'nals relative to the
movement are also relaye) to the basal n"clei@ re) n"cle"s@ an)
brainstem retic"lar <ormation.
o The basal n"clei eval"ate the comman) si'nal sent )own the
,yrami)al ,athways an) may contrib"te to the establishment o< nee)e)
bac9'ro"n) m"scle tone <or the movement. The n"clei are able to )o
this in ,art by ,roIectin' to the re) n"clei@ which in<l"ence s,inal cor)
al,ha an) 'amma motor ne"rons via r"bros,inal tracts.
o Similar in)irect ro"tin' to the s,inal cor) is achieve) thro"'h
corticoretic"los,inal an) corticor"bros,inal ,athways .
o The <"nction o< these in)irect ,athways to the s,inal cor) motor
ne"rons may incl")e more than ,rovi)in' bac9'ro"n) m"scle tone <or
movements )irecte) by the motor corte3.
o Even so@ beca"se the re) n"cle"s receives in,"t <rom the basal an)
cerebellar n"clei as well as )irect in,"t <rom the cerebral corte3@ its
<"nction may incl")e mo)i<yin' or U<ine t"nin'U the motor ne"rons which
innervate the m"scles involve) in a 'iven movement.
7ee)bac9 Loo,s
o The <ee)bac9 loo,s )escribe) here incl")e ne"ral circ"its in which a
si'nal sam,le is <e) bac9 to a Ucom,arator@U which is in a ,osition to
com,are the si'nal with some )esire) con)ition an) s"bse?"ently ta9e
ste,s to Ua)I"stU or Umo)i<yU it.
o The e3tra,yrami)al system incl")es two s"ch <ee)bac9 systemsR
*) cortically ori'inatin' e3tra,yrami)al system <ee)bac9 loo,s .%/EPS
feedbac+ loops0
+) ,ro,rioce,tor ori'inatin' e3tra,yrami)al system <ee)bac9 loo,s
.P/EPS feedbac+ loops0.
o The .O EBS <ee)bac9 loo,s are
*) com,ose) o< <ibers ori'inatin' in the motor corte3 which syna,se
in s"bcortical centers.
+) A<ter inte'ratin' an) eval"atin' the si'nals@ the centers ,roIect
<ibers bac9 to the cortical so"rce <or mo)i<ication.
:) 1n loo, A the si'nal is Uta,,e) o<<U to the cor,"s striat"m (ca")ate
an) ,"tamen)@ which in t"rn ,roIect to the 'lob"s ,alli)"s.
Balli)othalamic <ibers then ,roIect to the thalam"s@ which com,letes
the loo, by ,roIectin' bac9 to the cortical so"rce. Somewhere in this
loo, the ori'inal si'nal sent )own the ,yrami)al tracts is com,are)
an) eval"ate) with other in,"t relative to the movement. A<ter
a,,ro,riate inte'ration@ mo)i<yin' <ee)bac9 si'nals are ret"rne) to
the corte3 via the thalamocortical <ibers.
#) 1n loo, 4 the sam,le si'nal is sent to ,ontine n"clei <or s"bse?"ent
re<erral to the cerebell"m@ where it is ,robably com,are) to
,ro,rioce,tive in,"t comin' <rom m"scles@ ten)ons@ an) Ioints
involve) in the movement. This in,"t ,robably incl")es s"ch thin's
as the c"rrent state o< m"scle tone an) the relative ,osition an)
movement o< the limb involve). 7ollowin' inte'ration o< this in,"t@
the cerebell"m then ,roIects its o"t,"t to the thalam"s (via )entaG
tothalamic tracts) which then com,letes the loo, by sen)in' <ibers
bac9 to the cortical so"rce thro"'h thalamocortical ,roIections.
-) 1n loo, .. the sam,le si'nal is sent to the s"bstantia ni'ra. which
,roIects in t"rn to the cor,"s striat"m. 7rom here the <ee)bac9
circ"it is i)entical to that ill"strate) in loo, A.
o BOEBS <ee)bac9 loo,s.
*) 1n this system the mo)i<ication is not )irecte) bac9 towar) the
cortical so"rce (as are the .OEBS loo,s)@ b"t to the s,inal cor) motor
ne"rons instea).
+) The ,rinci,al loo, involves the relay o< m"scle@ ten)on. an) Ioint
,ro,rioce,tive in<ormation to the cerebell"m via the s,inocerebellar
tracts.
:) The si'nals are inte'rate) in the cerebell"m an) ,robably com,are)
with the inten)e) si'nals sam,le) by cortico,ontocerebellar ,athways.
#) 1t co"l) then )irect mo)i<ication thro"'h its ,roIections to the
vestib"lar. retic"lar@ an) r"bral n"clei an) their res,ective )escen)in'
tracts to the a,,ro,riate motor ne"rons o< the s,inal cor).
A")itory /is"al /estib"lar ;escen)in' Bathways
Bost"ral a)I"stments in res,onse to a")itory@ vis"al. an) vestib"lar
si'nals is an a))itional way to re'"late the activity o< s,inal motor
ne"rons.
A")itory an) vis"al in,"t to the tectal n"clei o< the mi)brain may be
res,onsible <or ,ro)"cin' re<le3 movements o< the bo)y in res,onse to a
s"))en so"n) or bri'ht li'ht.
Similarly. in,"t <rom the vestib"lar a,,arat"s to the vestib"lar n"clei
an) cerebell"m no )o"bt ,lays a role in re<le3 ,ost"ral a)I"stments
thro"'h the vestib"los,inal an) other tracts.
-ononeuropathy -ultipleF
*+FI'I!I)'
-ononeuropathy multipleF re<ers to the m"lti<ocal involvement o<
in)ivi)"al ,eri,heral nerves.
-ononeuritis multipleF more o<ten an in<lammatory ca"se is
res,onsible@ an) in s"ch cases the )isor)er is re<erre) to as
mononeuritis multiple$.
C1US+S
systemic (0&=) an) nonsystemic (::=) vasc"litis [ less commonly@
vasc"litic ne"ro,athy may ,resent as monone"ritis m"lti,le3E

Amon' the systemic vasc"litis@
1),olyarteritis no)osa(BAN)@
2)rhe"matoi) arthritis(RA)@
:) systemic l","s erythematos"s (SLE)@
#) .h"r'GStra"ss syn)rome@
-) De'enerFs 'ran"lomatosis@
0) hy,ersensitivity vasc"litis
these are o<ten associate) with constit"tional sym,toms s"ch as <ever an)
wei'ht loss .
vasc"litic ne"ro,athy
*) The common <ib"lar nerve is a<<ecte) in 8&-= o< ,atients withE
sym,toms consist o< a ,ain<"l <oot )ro,.
+) The "lnar@ me)ian@ an) ra)ial nerves may also be involve).
!reatment
Thera,y o< the systemic vasc"litis
can stabiliHe an) in some cases im,rove the ne"ro,athy.
l"cocorticoi)s L,re)nisone (*.- m'(9' ,er )ay)M ,l"s a cytoto3ic a'ent
("s"ally oral cyclo,hos,hami)e at + m'(9' ,er )ay) is the treatment o<
choice .
Bre)nisone can be chan'e) to an alternateG)ay re'imen a<ter * month to
minimiHe si)e e<<ects.
Once a clinical res,onse is )oc"mente)@ ,re)nisone may be ta,ere) by
- m' every +%# wee9s.
The cytoto3ic a'ent is "s"ally contin"e) <or * year.
Thera,y o< hy,ersensitivity vasc"litis
<oc"se) ,rimarily ",on removal o< the o<<en)in' anti'en tri''er.
Treatment o< localiHe) vasc"litis
restricte) to the ,eri,heral nervo"s system can be less a''ressive than
<or systemic vasc"litis beca"se the ris9 o< )eath <rom "ntreate) )isease
is very low.
>onothera,y with either oral 'l"cocorticoi)s or a brie< co"rse o<
cyclo,hos,hami)e (:%0 months) may be s"<<icient.
.
.uillainA5arrI Syndrome
;E71N1T1ON
"illainG4arr\ syn)rome (4S) is an ac"te@ <re?"ently severe@ an)
<"lminant ,olyra)ic"lone"ro,athy that is a"toimm"ne in nat"re
;ia'nostic .riteria <or "illainG4arr\ Syn)rome
(eHuired
*. Bro'ressive wea9ness o< + or more limbs )"e to
ne"ro,athy
+. Are<le3ia
:. ;isease co"rse 5# wee9s
*. E3cl"sion o< other ca"ses
vasc"litis (BAN@SLE@ .h"r'GStra"ss syn)rome)@
to3ins (or'ano,hos,hates@ lea))@
bot"lism@
)i,htheria@
,or,hyria@
localiHe) s,inal cor) or ca")a e?"ina syn)rome
Supportive
*. Relatively symmetric wea9ness
+. >il) sensory involvement
:. 7acial nerve or other cranial nerve involvement
#. Absence o< <ever
-. Ty,ical .S7 ,ro<ile (acell"lar@ increase in ,rotein level)
0. Electro,hysiolo'ic evi)ence o< )emyelination
Treatment
1n the vast maIority o< ,atients with 4S@ treatment sho"l) be initiate)
as soon a<ter )ia'nosis as ,ossible.
8+ wee9s a<ter the <irst motor sym,toms@ imm"nothera,y is no lon'er
e<<ective.
Either hi'hG)ose intraveno"s imm"ne 'lob"lin (1/1') or ,lasma,heresis
can be initiate)@ as they are e?"ally e<<ective.
1/1' is o<ten the initial thera,y chosen beca"se o< its ease o<
a)ministration an) 'oo) sa<ety recor). 1/1' is a)ministere) as <ive )aily
in<"sions <or a total )ose o< + '(9' bo)y wei'ht@ ,erha,s acco"ntin' <or
the thera,e"tic e<<ect.
A co"rse o< ,lasma,heresis "s"ally consists o< 8#$%-$ mL(9' ,lasma
e3chan'e (BE) <o"r times over a wee9.
treatment re)"ces the nee) <or mechanical ventilation by nearly hal< @
an) increases the li9elihoo) o< <"ll recovery at * year.
1n ,atients who are treate) early in the co"rse o< 4S an) im,rove@
rela,se may occ"r in the secon) or thir) wee9. 4rie< retreatment with
the ori'inal thera,y is "s"ally e<<ective.
l"cocorticoi)s have not been <o"n) to be e<<ective in 4S.
Occasional ,atients with very mil) <orms o< 4S@ es,ecially those who
a,,ear to have alrea)y reache) a ,latea" when initially seen@ may be
mana'e) conservatively witho"t 1/1' or BE.
1n the worsenin' ,hase o< 4S @ most ,atients re?"ire monitorin' in a
critical care settin'@ with ,artic"lar attention to
*. vital ca,acity@
+. heart rhythm@
:. bloo) ,ress"re@
#. n"trition@
-. )ee, vein thrombosis ,ro,hyla3is@
0. car)iovasc"lar stat"s@
&. early consi)eration (a<ter + wee9s o< int"bation) o< tracheotomy@
6. chest ,hysiothera,y.
As note)@ 8:$= o< ,atients with 4S re?"ire ventilatory assistance@
sometimes <or ,rolon'e) ,erio)s o< time (several wee9s or lon'er).
7re?"ent t"rnin' an) assi)"o"s s9in care are im,ortant@ as are )aily
ran'eGo<Gmotion e3ercises to avoi) Ioint contract"res
)aily reass"rance as to the 'enerally 'oo) o"tloo9 <or recovery.
-/+S!2+'I1
.(1@IS
I'!()*UC!I)'
>yasthenia 'ravis (>) is a neuromuscular disorder
characteriHe) by wea9ness an) <ati'ability o< s9eletal m"scles.
!he underlying defect is a )ecrease in the n"mber o< available
acetylcholine rece,tors (A.hRs) at ne"rom"sc"lar I"nctions )"e
to an antibo)yGme)iate) a"toimm"ne attac9.
CLI'IC1L F+1!U(+S
> is not rare@ havin' a prevalence of 1$4 in 10=000.
1t a<<ects in)ivi)"als in all age groups@ b"t pea<s of incidence
occ"r in women in their twenties [ thirties an) in men in their
<i<ties [ si3ties. (+$G:$(-$G0$)
Overall@ >omen are affected more freHuently than men@ in a
ratio o< 8:R+.
!he course of -. is o<ten variable. +Facerbations and
remissions may occ"r@ ,artic"larly )"rin' the <irst <ew years
a<ter the onset o< the )isease.
Remissions are rarely com,lete or ,ermanent.
The cardinal features are -ea+ness an) fatigability o< m"scles.
The >ea<ness increases )"rin' re,eate) "se (<ati'"e) an) may
improve <ollowin' rest or slee,.
!he distribution of muscle >ea<ness o<ten has a
characteristic ,attern.
*. !he cranial muscles= particularly the lids and
eFtraocular muscles= are o<ten involve) early in the co"rse o<
>@ an) )i,lo,ia an) ,tosis are common initial com,laints.
+. Facial >ea<ness ,ro)"ces a Usnarlin'U e3,ression when the
,atient attem,ts to smile.
:. ?ea<ness in che>ing is most noticeable a<ter ,rolon'e)
e<<ort@ as in chewin' meat.
#. Speech may have a nasal timbre ca"se) by wea9ness o< the
,alate or a )ysarthric Um"shyU ?"ality )"e to ton'"e wea9ness.
-. *ifficulty in s>allo>ing may occ"r as a res"lt o< wea9ness
o< the ,alate@ ton'"e@ or ,haryn3@ 'ivin' rise to nasal
re'"r'itation or as,iration o< li?"i)s or <oo).
In J"#% of patients@ the wea9ness becomes 'eneraliHe)@
a<<ectin' the limb m"scles as well.
!he limb >ea<ness in -. is o<ten ,ro3imal an) may be
asymmetric.
;es,ite the m"scle wea9ness@ deep tendon refleFes are
,reserve).
If >ea<ness remains restricted to the eFtraocular muscles
for , years= it is li9ely that it will not become 'eneraliHe)@ an)
these ,atients are sai) to have ocular M1.
-otor po>er survey G ?"antitative testin' o< m"scle stren'th
For>ard arm abduction time (- min)
1bsence of other neurologic signs
I'@+S!I.1!I)'S
3 Anti$"dies t" ACR "r MuSK
As note) above@ antiA1Ch( antibodies are )etectable in the
ser"m o< J"#% of all myasthenic patients
The presence of antiA1Ch( antibodies is virtually
diagnostic of -.= b"t a ne'ative test )oes not e3cl")e the
)isease.
1ntibodies to -uSD have been <o"n) to be ,resent in 8#$= o<
A.hR antibo)yGne'ative ,atients with 'eneraliHe) >@ an) their
,resence is a "se<"l )ia'nostic test in these ,atients.
4 Electr"diagn"stic Testing
(epetitive nerve stimulation o<ten ,rovi)es hel,<"l )ia'nostic
evi)ence o< >.
*. 1ntiA1Ch+ medication is sto,,e) 0%+# h be<ore testin'.
+. It is $est t" test wea9 m"scles or ,ro3imal m"scle 'ro",s.
:. +lectric shoc<s are delivered at a rate o< two or three ,er
secon) to the a,,ro,riate nerves@ an) action potentials are
recorded <rom the m"scles.
#. In normal individuals@ the am,lit")e o< the(E>AB) evo9e)
m"scle action ,otentials )oes not chan'e at these rates o<
stim"lation.
-. However= in myasthenic patients there is a ra,i) re)"ction
o< C*$%*-= in the am,lit")e o< the evo9e) res,onses.
0. 1s a further test= a single dose of edrophonium may be
gi&en to pre&ent or diminish this decremental response.
5 Antic"linesterase Test
!he edrophonium test is no> reserved for ,atients with
clinical <in)in's that are s"''estive o< > b"t who have ne'ative
antibo)y an) electro)ia'nostic test res"lts.
+drophonium is used most commonly for diagnostic
testing
In some cases it is helpful to use a longerAacting drug such
as neostigmine (*- m' BO)@ since this ,ermits more time <or
)etaile) eval"ation o< stren'th.
6 Single7fi$er electr"#%"gra'%2
bloc<ing and Kitter@ with normal <iber )ensityE
confirmatory@ b"t not s,eci<ic
8 F"r "cular "r cranial MG2
eFclude intracranial lesions by .T or >R1
9 Rec"##ended la$"rat"r% tests "r 'r"cedures t"
guide trat#ent
*. .T or >R1 o< me)iastin"m ( thymoma@ hy,er,lasia)
+. Tests <or l","s erythematos"s@ antin"clear antibo)y@
rhe"matoi) <actor@ antithyroi) antibo)ies
:. Thyroi)G<"nction tests (T7T)
#. BB; s9in test (occ"lt in<ection)
-. .hest ra)io'ra,hy (tb)
0. 7astin' bloo) 'l"cose meas"rement@ hemo'lobin A*c (;>)
&. B"lmonaryG<"nction tests (asthama)
6. 4one )ensitometry in ol)er ,atients(Osteo,oresis)
!(+1!-+'!
+irtuall% all #%astenic 'atients can $e returned t" full
'r"ducti!e li!es .it 'r"'er tera'%,
3, Antic"linesterase Medicati"ns
Pyridostigmine is the most wi)ely "se) anticholinesterase )r"'.
Treatment is begun >ith a moderate dose@ e.'.@ :$%0$ m'
three to <o"r times )aily.
!he freHuency and amount of the dose should be tailored
to the ,atientFs in)ivi)"al re?"irements thro"'ho"t the )ay. 7or
e3am,le@ patients >ith >ea<ness in che>ing and
s>allo>ing may bene<it by ta9in' the me)ication be<ore meals
so that ,ea9 stren'th coinci)es with mealtimes.
4, T%#ect"#%
1n the absence o< a t"mor@ the available evidence suggests
that ", to 6-= o< ,atients e3,erience im,rovement a<ter
thymectomyE
It is the consensus that thymectomy should be carried out
in all ,atients with 'eneraliHe) > who are between the a'es o<
,"berty an) at least -- years.
5, I##un"su''ressi"n
For the intermediate term@ 'l"cocorticoi)s an) cyclos,orine or
tacrolim"s 'enerally ,ro)"ce clinical im,rovement within a ,erio)
o< *%: months.
for the longAterm treatment of patients with >. The
bene<icial e<<ects o< aHathio,rine an) myco,henolate mo<etil
Gluc"c"rtic"id Tera'%
!he initial dose should be relatively lo> (*-%+- m'())
!he dose is increased step>ise= as tolerated by the patient
("s"ally by - m'() at +G to :G)ay intervals)@ "ntil there is mar9e)
clinical im,rovement or a )ose o< -$%0$ m'() is reache).
!his dose is maintained for 1$, months an) then is
gradually modified to an alternateAday regimen over the
course of an additional 1$, monthsE
Oter I##un"su''ressi!e Drugs
M%c"'en"late #"fetil
1 dose of 1$1&# g bid is recommen)e).
!his drug has become the choice for longAterm treatment
o< myasthenic ,atients.
Un<ort"nately@ the cost of mycophenolate is still very high
A:AT/IO;RENE
1n initial dose of #0 mgd sho"l) be "se) to test <or a)verse
si)e e<<ects.
1< this )ose is tolerate)@ it is increase) 'ra)"ally "ntil the white
bloo) co"nt <alls to 8:$$$%#$$$("L.
The typical dosage range is 2$, mg<g total bo)y wei'ht.
C%cl"s'"rine and tacr"li#us
!he usual dose of cyclosporine is #%- m'(9' ,er )ay@
The average dose of tacrolimus is $.* m'(9' ,er )ay@ 'iven in
t-o e2ually di&ided doses
Side effects of these drugs incl")e hy,ertension an)
ne,hroto3icity@ which m"st be closely monitore).
C%cl"'"s'a#ide
a co"rse o< highAdose cyclophosphamide may in)"ce lon'G
lastin' (,ossibly ,ermanent) bene<it by Urebootin'U the imm"ne
system.
6, ;las#a'eresis
1 course of five eFchanges 7:%# L ,er e3chan'e) is 'enerally
a)ministere) over a 34, to 35,day period.
Plasmapheresis produces a shortAterm reduction in antiG
A.hR antibo)ies@ with clinical im,rovement in many ,atients.
It is useful as a temporary eFpedient in serio"sly a<<ecte)
,atients or to im,rove the ,atientFs con)ition ,rior to s"r'ery
(e.'.@ thymectomy).
8, Intra!en"us I##un"gl"$ulin
!he indications for the use of I@Ig are the same as those
for plasma eFchange
!his treatment has the advantages o< not re?"irin' s,ecial
e?"i,ment or lar'eGbore veno"s access.
!he usual dose is 2 g<g@ which is ty,ically a)ministere) over -
)ays (#$$ m'(9' ,er )ay).
-anagement of -yasthenic Crisis
-yasthenic crisis is defined as an e3acerbation o< wea9ness
s"<<icient to en)an'er li<eE it usually consists o< res,iratory
<ail"re ca"se) by )ia,hra'matic an) intercostal m"scle wea9ness.
Crisis rarely occurs in ,ro,erly mana'e) ,atients.
!reatment should be carried out in intensive care units
The ,ossibility that )eterioration co"l) be due to eFcessive
anticholinesterase medication 7Lcholinergic crisisL9 is best
e3cl")e) by tem,orarily sto,,in' anticholinesterase )r"'s.
The most common ca"se o< crisis is intercurrent infection.This
should be treated immediately
!he myasthenic patient >ith fever and early infection
sho"l) be treate) li9e other imm"nocom,romise) ,atients.
+arly and effective antibiotic therapy= respiratory
assistance= and pulmonary physiotherapy are essentials o<
the treatment ,ro'ram.
As )isc"sse) above@ plasmapheresis or I@Ig is freHuently
helpful in hastening recovery.
*rugs to 1void in -yasthenic
Patients
As a rule& te listed drugs s"uld $e a!"ided
.ene!er '"ssi$le& and #%astenic 'atients s"uld
$e f"ll".ed cl"sel% .en an% ne. drug is intr"duced,
*rugs !hat -ay +Facerbate -. ( A4 L>N B])
1ntibiotics
Amino'lycosi)esR e.'.@ stre,tomycin@ tobramycin@ 9anamycin
]"inolonesR e.'.@ ci,ro<lo3acin@ levo<lo3acin@ o<lo3acin@ 'ati<lo3acin
>acroli)esR e.'.@ erythromycin@ aHithromycin@
5etaAbloc<ing agents
Bro,ranalol@ atenolol@ meto,rolol
5otulinum toFin
4oto3 e3acerbates wea9ness
Local anesthetics and related agents
Brocaine@
3ylocaine in lar'e amo"nts
Brocainami)e (<or arrhythmias)
-agnesium
'ondepolariCing muscle relaFants for surgery
;GT"boc"rarine (c"rare)@
,anc"roni"m@ vec"roni"m@ atrac"ri"m
Penicillamine
;uinine derivatives
]"inine@ ?"ini)ine@ chloro?"ine@ me<lo?"ine
*rugs >ith Important Interactions in -.
Cyclosporine
1Cathioprine Avoi) allo,"rinolNcombination may
res"lt in myelos",,ression.
.eneraliCed= !onicAClonic SeiCures
7.rand -al9
I'!()
Brimary 'eneraliHe)@ tonicGclonic seiH"res are the main seiH"re ty,e in
8*$= o< all ,ersons with e,ile,sy.
They are also the most common seiH"re ty,e res"ltin' <rom metabolic
)eran'ements an) are there<ore <re?"ently enco"ntere) in many
)i<<erent clinical settin's.
CLI'IC1L F+1!U(+
The seiH"re "s"ally be'ins abr",tly witho"t warnin'@ altho"'h some
,atients )escribe va'"e ,remonitory sym,toms in the ho"rs lea)in' ",
to the seiH"re.
The initial ,hase o< the seiH"re is "s"ally tonic contraction o< m"scles
thro"'ho"t the bo)y@ acco"ntin' <or a n"mber o< the classic <eat"res o<
the event.
*) Tonic contraction o< the m"scles o< e3,iration an) the laryn3 at
the onset will ,ro)"ce a lo") moan or Uictal cry.U
+) Res,irations are im,aire)@ secretions ,ool in the oro,haryn3@ an)
cyanosis )evelo,s.
:) .ontraction o< the Iaw m"scles may ca"se bitin' o< the ton'"e.
#) A mar9e) enhancement o< sym,athetic tone lea)s to increases in
heart rate@ bloo) ,ress"re@ an) ,",illary siHe.
A<ter *$%+$ s@ the tonic ,hase o< the seiH"re ty,ically evolves into the
clonic ,hase@ ,ro)"ce) by the s",erim,osition o< ,erio)s o< m"scle
rela3ation on the tonic m"scle contraction.
The ,erio)s o< rela3ation ,ro'ressively increase "ntil the en) o< the ictal
,hase@ which "s"ally lasts no more than * min.
The ,ostictal ,hase is characteriHe) by
*) "nres,onsiveness@
+) m"sc"lar <lacci)ity@
:) e3cessive salivation that can ca"se stri)oro"s breathin' an)
,artial airway obstr"ction.
#) 4la))er or bowel incontinence may occ"r at this ,oint.
Batients 'ra)"ally re'ain conscio"sness over min"tes to ho"rs@ an)
)"rin' this transition there is ty,ically a ,erio) o< ,ostictal con<"sion.
Batients s"bse?"ently com,lain o< hea)ache@ <ati'"e@ an) m"scle ache
that can last <or many ho"rs.
The )"ration o< im,aire) conscio"sness in the ,ostictal ,hase can be
e3tremely lon'@ i.e.@ many ho"rs@ in ,atients with ,rolon'e) seiH"res or
"n)erlyin' .NS )iseases s"ch as alcoholic cerebral atro,hy.
There are many variants o< the 'eneraliHe) tonicGclonic seiH"re@ incl")in'
,"re tonic an) ,"re clonic seiH"res.
4rie< tonic seiH"res lastin' only a <ew secon)s are es,ecially noteworthy
since they are "s"ally associate) with s,eci<ic e,ile,tic syn)romes
havin' mi3e) seiH"re ,henoty,es@ s"ch as the Lenno3Gasta"t syn)rome
I'@+S!I.1!I)'S
Laboratory St")ies
1. Ro"tine bloo) st")ies are in)icate) to i)enti<y the more common
metabolic ca"ses o< seiH"res@ s"ch as abnormalities in electrolytes@
'l"cose@ calci"m@ or ma'nesi"m@ an) he,atic or renal )isease.
2. A screen <or to3ins in bloo) an) "rine sho"l) also be obtaine) <rom all
,atients in a,,ro,riate ris9 'ro",s@ es,ecially when no clear
,reci,itatin' <actor has been i)enti<ie).
3. A l"mbar ,"nct"re is in)icate) i< there is any s"s,icion o< menin'itis or
ence,halitis@ an) it is man)atory in all ,atients in<ecte) with H1/@ even in
the absence o< sym,toms or si'ns s"''estin' in<ection.
Electro,hysiolo'ic St")ies
*. All ,atients who have a ,ossible seiH"re )isor)er sho"l) be eval"ate)
with an EE as soon as ,ossible.
2. The EE )"rin' the tonic ,hase o< the seiH"re shows a ,ro'ressive
increase in 'eneraliHe) lowGvolta'e <ast activity@ <ollowe) by 'eneraliHe)
hi'hGam,lit")e@ ,olys,i9e )ischar'es.
3. 1n the clonic ,hase@ the hi'hGam,lit")e activity is ty,ically interr",te) by
slow waves to create a s,i9eGan)Gwave ,attern.
4. The ,ostictal EE shows )i<<"se slowin' that 'ra)"ally recovers as the
,atient awa9ens.
4rain 1ma'in'
*. Almost all ,atients with newGonset seiH"res sho"l) have a brain
ima'in' st")y to )etermine whether there is an "n)erlyin' str"ct"ral
abnormality that is res,onsible.
2. >R1 has been shown to be s",erior to .T <or the )etection o< cerebral
lesions associate) with e,ile,sy.
3. 7"nctional ima'in' ,roce)"res s"ch as ,ositron emission tomo'ra,hy
(BET) an) sin'le ,hoton emission com,"te) tomo'ra,hy (SBE.T) are
also "se) to eval"ate certain ,atients with me)ically re<ractory seiH"res
-1'1.+-+'!
Thera,y <or a ,atient with a seiH"re )isor)er is almost always m"ltimo)al
an) incl")es
*) treatment o< "n)erlyin' con)itions@
+) avoi)ance o< ,reci,itatin' <actors@
:) s",,ression o< rec"rrent seiH"res by ,ro,hylactic thera,y with
antie,ile,tic me)ications or s"r'ery@
#) a))ressin' a variety o< ,sycholo'ical an) social iss"es.
Treatment o< Un)erlyin' .on)itions
*) 1< the sole ca"se o< a seiH"re is a metabolic )ist"rbance s"ch as
an abnormality o< ser"m electrolytes or 'l"cose@ then treatment is
aime) at reversin' the metabolic ,roblem an) ,reventin' its
rec"rrence.
+) 1< the a,,arent ca"se o< a seiH"re was a me)ication (e.'.@
theo,hylline) or illicit )r"' "se (e.'.@ cocaine)@ then a,,ro,riate
thera,y is avoi)ance o< the )r"'
:) SeiH"res ca"se) by a str"ct"ral .NS lesion s"ch as a brain t"mor@
vasc"lar mal<ormation@ or brain abscess may not rec"r a<ter
a,,ro,riate treatment o< the "n)erlyin' lesion.
Avoi)ance o< Breci,itatin' 7actors
*) a ,atient who has seiH"res in the settin' o< slee, )e,rivation
sho"l) obvio"sly be a)vise) to maintain a normal slee, sche)"le.
+) >any ,atients note an association between alcohol inta9e an)
seiH"res@ an) they sho"l) be enco"ra'e) to mo)i<y their )rin9in'
habits accor)in'ly.
:) There are also relatively rare cases o< ,atients with seiH"res that
are in)"ce) by hi'hly s,eci<ic stim"li s"ch as a vi)eo 'ame monitor@
m"sic@ or an in)ivi)"alFs voice (Ure<le3 e,ile,syU).
#) 1< there is an association between stress an) seiH"res@ stress
re)"ction techni?"es s"ch as ,hysical e3ercise@ me)itation@ or
co"nselin' may be hel,<"l.
Dhen to 1nitiate Antie,ile,tic ;r"' Thera,y
*) ,atient with rec"rrent seiH"res o< "n9nown etiolo'y
+) a 9nown ca"se that cannot be reverse).
Antie,ile,tic ;r"' Selection <or eneraliHe) SeiH"res
*) /al,roic aci) an) lamotri'ine are c"rrently consi)ere) the best
initial choice <or the treatment o< ,rimary 'eneraliHe)@ tonicGclonic
seiH"res.
+) Bhenytoin@ <ollowe) by to,iramate@ carbamaHe,ine@ an)
Honisami)e are s"itable alternatives.
@alproic acid 20$30 mg<gM bidAHid
Lamotrigine 1#0$#00 mgdM bid
Phenytoin ,$3 mg<g= adultM 0$" mg<g= childM HdAbid
Dhen to ;iscontin"e Thera,y
o Overall@ abo"t &$= o< chil)ren an) 0$= o< a)"lts who have their
seiH"res com,letely controlle) with antie,ile,tic )r"'s can event"ally
)iscontin"e thera,y.
Treatment o< Re<ractory E,ile,sy
o A,,ro3imately oneGthir) o< ,atients with e,ile,sy )o not res,on) to
treatment with a sin'le antie,ile,tic )r"'@ an) it becomes necessary
to try a combination o< )r"'s to control seiH"res
S!1!US
+PIL+P!ICUS
;E71N1T1ON
Stat"s e,ile,tic"s re<ers to contin"o"s seiH"res or
re,etitive@ )iscrete seiH"res with im,aire)
conscio"sness in the interictal ,erio).
The )"ration o< seiH"re activity s"<<icient to meet the
)e<inition o< stat"s e,ile,tic"s has tra)itionally been
s,eci<ie) as *-%:$ min.
However@ a more ,ractical )e<inition is to consi)er
stat"s e,ile,tic"s as a sit"ation in which the )"ration o<
seiH"res ,rom,ts the ac"te "se o< anticonv"lsant
thera,y. 7or .SE@ this is ty,ically when seiH"res last
beyon) - min.
SU4GT!BES
Stat"s e,ile,tic"s has n"mero"s s"bty,es@ incl")in'
'eneraliHe) conv"lsive stat"s e,ile,tic"s (.SE)
*. ,ersistent 'eneraliHe) electro'ra,hic seiH"res@
+. ,ersistent coma@
:. ,ersistent tonicGclonic movements@
nonconv"lsive stat"s e,ile,tic"s
*. ,ersistent absence seiH"res
+. ,ersistent ,artial seiH"res@
:. ,ersistent con<"sion
#. ,ersistent ,artially im,aire) conscio"sness@ an)
-. ,ersistent minimal motor abnormalities.
.SE
The most common ca"ses o< .SE are
*. re<ractory e,ile,sy@
+. )r"' to3icity
:. anticonv"lsant with)rawal or noncom,liance@
#. metabolic )ist"rbances@
-. .NS in<ection@
0. .NS t"mors@
&. hea) tra"ma.
.L1N1.AL
.SE is obvio"s when the ,atient is havin' overt
conv"lsions.
However@ a<ter :$%#- min o< "ninterr",te) seiH"res@ the
si'ns may become increasin'ly s"btle. Batients may
have mil) clonic movements o< only the <in'ers or <ine@
ra,i) movements o< the eyes.
There may be ,aro3ysmal e,iso)es o< tachycar)ia@
hy,ertension@ an) ,",illary )ilation. 1n s"ch cases@ the
EE may be the only metho) o< establishin' the
)ia'nosis.
Th"s@ i< the ,atient sto,s havin' overt seiH"res@ yet
remains comatose@ an EE sho"l) be ,er<orme) to r"le
o"t on'oin' stat"s e,ile,tic"s.
>ANAE>ENT
1%SE is an emergency and must be treated
immediately
The <irst ste, in the mana'ement o< a ,atient in .SE is
*. to atten) to any ac"te car)iores,iratory ,roblems
or hy,erthermia@
+. ,er<orm a brie< me)ical an) ne"rolo'ic
e3amination@
:. establish veno"s access@
#. sen) sam,les <or laboratory st")ies to i)enti<y
metabolic abnormalities.
Anticonv"lsant thera,y sho"l) then be'in witho"t
)elayE a treatment a,,roach is shown in harrison <i'
:0:G:.
The treatment o< nonconv"lsive stat"s e,ile,tic"s is
somewhat less "r'ent than .SE@ since the on'oin'
seiH"res are not accom,anie) by the severe metabolic
)ist"rbances seen with .SE.
However@ evi)ence s"''ests that nonconv"lsive stat"s
e,ile,tic"s@ es,ecially that ca"se) by on'oin'@ <ocal
seiH"re activity@ is associate) with cell"lar inI"ry in the
re'ion o< the seiH"re <oc"s@ so that the con)ition sho"l)
be treate) as ,rom,tly as ,ossible "sin' the 'eneral
a,,roach )escribe) <or .SE.
5ro>nASIHuard syndrome
1NTRO
4rownGS\?"ar) syn)rome is a loss o< sensation an) motor <"nction that
is ca"se) by the lateral hemisection o< the s,inal cor).
.lassi<ication
Any ,resentation o< s,inal inI"ry that is an incom,lete lesion can be
calle) a ,artial 4rownGS\?"ar) or incom,lete 4rownGS\?"ar) syn)rome@
so lon' as it is characteriHe) by a motor )e<icit an) n"mbness to to"ch
an) vibration on the same si)e o< the s,inal inI"ry an) loss o< ,ain an)
tem,erat"re sensation on the o,,osite si)e
Causes
4rownGS\?"ar) syn)rome may be ca"se) by
*. a s,inal cor) t"mo"r@
+. tra"ma (s"ch as a '"nshot wo"n) or ,"nct"re wo"n) to the nec9
or bac9)@
:. ischemia (obstr"ction o< a bloo) vessel)@ or
#. in<ectio"s or in<lammatory )iseases s"ch as t"berc"losis@ or
m"lti,le sclerosis.
The most common ca"se is ,enetratin' tra"ma s"ch as a '"nshot wo"n)
or stab wo"n) to the s,inal cor). This may be seen most o<ten in the
cervical (nec9) or thoracic s,ine.
Batho,hysiolo'y [ .L1N1.AL BRESENTAT1ON
4rownGS\?"ar) syn)romeFs sym,tomsR
^ A Si)e o< the lesion
* A hy,ertonic ,aralysis
+ A s,astic ,aralysis an) loss o< vibration an) ,ro,rioce,tion (,osition
sense) an) <ine to"ch
: A loss o< ,ain an) tem,erat"re sensation
As a res"lt o< the inI"ry to three main brain ,athways the ,atient will
,resent with three lesionsR
*. The corticos,inal lesion ,ro)"ces s,astic ,aralysis on the same si)e
o< the bo)y (the loss o< mo)eration by the U>N).
+. The lesion to <ascic"l"s 'racilis or <ascic"l"s c"neat"s res"lts in
i,silateral loss o< vibration an) ,ro,rioce,tion (,osition sense) as well as
loss o< all sensation o< <ine to"ch.
:. The loss o< the s,inothalamic tract lea)s to ,ain an) tem,erat"re
sensation bein' lost <rom the contralateral si)e be'innin' one or two
se'ments below the lesion.
*iagnosis
>a'netic resonance ima'in' (>R1) is the ima'in' o< choice in s,inal cor)
lesions.
!reatment
Treatment is )irecte) at the ,atholo'y ca"sin' the ,aralysis.
1< it is beca"se o< tra"ma s"ch as a '"nshot or 9ni<e wo"n)@ there may
be other li<e threatenin' con)itions s"ch as blee)in' or maIor or'an
)ama'e which sho"l) be )ealt with on an emer'ent basis.
1< the syn)rome is ca"se) by a s,inal <ract"re@ this sho"l) be i)enti<ie)
an) treate) a,,ro,riately.
Altho"'h steroi)s may be "se) to )ecrease cor) swellin' an)
in<lammation@ the "s"al thera,y <or s,inal cor) inI"ry is e3,ectant
Subacute Sclerosing
Panencephalitis 7SSP+9
*+FI'I!I)'
SSBE is a rare chronic@ ,ro'ressive )emyelinatin' )isease o< the .NS
associate) with a chronic non,ermissive in<ection o< brain tiss"e with
measles vir"s.
I'CI*+'C+
The inci)ence has )ecline) )ramatically since the intro)"ction o< a
measles vaccine.
CLI'IC1L F+1!U(+S
>ost ,atients 'ive a history o< ,rimary measles in<ection at an early a'e
(+ years)@ which is <ollowe) a<ter a latent interval o< 0%6 years by the
)evelo,ment o< ,ro'ressive ne"rolo'ic )isor)er.
,atients are between - an) *- years ol) at )ia'nosis.
1nitial mani<estations incl")e
*) ,oor school ,er<ormance
+) moo) an) ,ersonality chan'es.
Ty,ical si'ns o< a .NS viral in<ection@ incl")in' <ever an) hea)ache@ )o
not occ"r.
As the )isease ,ro'resses @ ,atients )evelo,
*) ,ro'ressive intellect"al )eterioration@
+) <ocal an)(or 'eneraliHe) seiH"res@
:) myoclon"s@
#) ata3ia@
-) vis"al )ist"rbances.
1n the late sta'e o< the illness @ ,atients are
*) "nres,onsive@
+) s,astic ?"a)ri,aretic@
:) hy,eractive ten)on re<le3es
#) e3tensor ,lantar res,onses.
*iagnostic Studies
>R1 is o<ten normal early@ altho"'h areas "f increased T4 signal
de!el"' in te .ite #atter "f te $rain and $rainste# as
disease 'r"gresses.
The EE may initially show only n"ns'ecific sl".ing@ b"t with )isease
,ro'ression@ ,atients )evelo, a characteristic 'eri"dic 'attern .it
$ursts "f ig7!"ltage& sar'& sl". .a!es e!er% 5<= s& f"ll".ed
$% 'eri"ds "f attenuated (>flat>* $ac?gr"und,
The .S7 is
*) acell"lar
+) normal or mil)ly elevate) ,rotein concentration
:) a mar9e)ly elevate) 'amma 'lob"lin level (C+$= o< total .S7
,rotein).
4).S7 antiGmeasles antibo)y levels are invariably elevate)@
5)oli'oclonal antiGmeasles antibo)ies are o<ten ,resent.
Measles !irus can be c"lt"re) <rom brain tiss"e "sin' s,ecial
c"culti!ati"n tecni@ues.
+iral antigen can be i)enti<ie) imm"nocytochemically@
!iral gen"#e can be )etecte) by in sit" hybri)iHation or B.R
am,li<ication.
!reatment
No )e<initive thera,y <or SSBE is available.
Treatment with iso,rinosine (*$$ m'(9' ,er )ay)@ alone or in
combination with intrathecal or intraventric"lar al,ha inter<eron@ has
been re,orte) to ,rolon' s"rvival an) ,ro)"ce clinical im,rovement in
some ,atients
?ernic<e encephalopathy
I'!()
?ernic<e encephalopathy is a syn)rome characterise) by
*) o,hthalmo,le'ia @
+) ata3ia @
:) con<"sion@
#) im,airment o< shortGterm memory

Cause
*. thiamine (vitamin 4
*
) )e<iciency
+. ,rolon'e) alcohol@ am,hetamine cons"m,tion res"ltin' in thiamine
)e<iciency.
:. 'astric )isor)ers as carcinoma@ chronic 'astritis@
#. .rohnFs )isease@
-. re,etitive vomitin'@ ,artic"larly a<ter bariatric s"r'ery.
Presentation
Dernic9eFs ence,halo,athy be'ins abr",tly@ "s"ally with
*) eye movement )isor)ers (nysta'm"s@ 'aHe ,alsies@ an)
o,hthalmo,le'ia@ es,ecially o< the lateral rect"s m"scles)@
+) 'ait ata3ia @
:) con<"sion @
#) con<ab"lation @
-) shortGterm memory loss.
The classic tria) o< the syn)rome is
*) ence,halo,athy (brain )ama'e)@
+) o,hthalmo,le'ia (eye ,aralysis)@
:) ata3ia (loss o< coor)ination).
Untreate)@ it may ,ro'ress to Korsa9o<<Fs ,sychosis@ coma an) )eath.
!reatment
Treatment be'ins with intraveno"s or intram"sc"lar inIection o<
thiamine@ <ollowe) by assessment o< central nervo"s system an)
metabolic con)itions.
1n the ,resence o< s"bGclinical thiamine )e<iciency@ a lar'e )ose o< s"'ar
(es,ecially 'l"cose) can ,reci,itate the onset o< overt ence,halo,athyE
there<ore@ correctin' hy,o'lycemia sho"l) not be attem,te) be<ore
thiamine re,lenishment.
Rehy)ration to restore bloo) vol"me sho"l) <ollow@ as nee)e).
Dhen treate) early@ recovery may be ra,i) an) com,leteE tho"'h there
are almost always some minor ne"rolo'ical si'ns that ,ersist
!he 1lcohol ?ithdra>al Syndrome
I'!()
Once the brain has been re,eate)ly e3,ose) to hi'h )oses o< alcohol@
any s"))en )ecrease in inta9e can ,ro)"ce with)rawal sym,toms@ many
o< which are the o,,osite o< those ,ro)"ce) by into3ication.
CLI'IC1L F+1!U(+S
4eca"se alcohol has a short hal<Gli<e@ these with)rawal sym,toms
'enerally be'in within -%*$ h o< )ecreasin' ethanol inta9e@ ,ea9 in
intensity on )ay + or :@ an) im,rove by )ay # or -.
Features include
*) tremor o< the han)s (sha9es or Iitters)E
+) a'itation an) an3ietyE
:) a"tonomic nervo"s system overactivity incl")in' an increase in
,"lse@ res,iratory rate@ an) bo)y tem,erat"reE
#) insomnia@ sometimes accom,anie) by <ri'htenin' )reams.
5) + TO -= o< alcoholics e3,erience with)rawal seiH"res@ o<ten within
#6 h o< sto,,in' )rin9in'
1nFiety= insomnia= and mild levels of autonomic dysfunction may
,ersist to some )e'ree <or #%0 months as a ,rotracte) abstinence
syn)rome@ which may contrib"te to the ten)ency to ret"rn to )rin9in'.
!he term deliriu# tre#ens 7*!s9 re<ers to an "ncommon state o<
intense ac"te with)rawal that incl")es
*. )eliri"m (mental con<"sion@ a'itation@ an) <l"ct"atin' levels o<
conscio"sness)
+. tremor
3. A"tonomic overactivity (e.'.@ mar9e) increases in ,"lse@ bloo)
,ress"re@ an) res,irations).
!(+1!-+'!
!he first step is to ,er<orm a thoro"'h ,hysical e3amination in all
alcoholics who are consi)erin' sto,,in' )rin9in'@ incl")in' a search <or
*. evi)ence o< liver <ail"re@
+. 'astrointestinal blee)in'@
:. car)iac arrhythmia@
#. in<ection@
-. 'l"cose or electrolyte imbalance.
!he second step is to o<<er reass"rance that the ac"te with)rawal is
short live) an) to o<<er a)e?"ate n"trition an) rest.
*. All ,atients sho"l) be 'iven oral m"lti,le 4 vitamins@ incl")in'
-$%*$$ m' o< thiamine )aily <or a wee9 or more.
+. 4eca"se most alcoholics who enter with)rawal are either
normally hy)rate) or mil)ly overhy)rate)@ 1/ <l"i)s sho"l) be
avoi)e) "nless there is evi)ence o< si'ni<icant recent blee)in'@
vomitin'@ or )iarrhea.
:. >e)ications can "s"ally be a)ministere) orally.
!he third step in treatment is to reco'niHe that most with)rawal
sym,toms are ca"se) by the ra,i) removal o< a .NS )e,ressant@ in this
case@ alcohol.
*. Dhile most .NS )e,ressants are e<<ective@ benHo)iaHe,ines have
the hi'hest mar'in o< sa<ety an) lowest cost an) are@ there<ore@ the
,re<erre) class o< )r"'s.
+. 4enHo)iaHe,ines with short hal<Glives are es,ecially "se<"l <or
,atients with serio"s liver im,airment or evi)ence o< ,ree3istin'
ence,halo,athy or brain )ama'e.
!reatment of the patient >ith *!s can be challen'in'@ an) the
con)ition is li9ely to r"n a co"rse o< :%- )ays re'ar)less o< the thera,y
em,loye).
*. The <oc"s o< care is to i)enti<y an) correct me)ical ,roblems an)
to control behavior an) ,revent inI"ries.
+. >any clinicians recommen) the "se o< hi'h )oses o< a
benHo)iaHe,ine (as m"ch as 6$$ m'() o< chlor)iaHe,o3i)e)@
3. Other clinicians recommen) the "se o< anti,sychotic me)ications@
s"ch as halo,eri)ol@ Or olanHa,ine
.eneraliCed >ithdra>al seiCures rarely re?"ire a''ressive
,harmacolo'ic intervention beyon) that 'iven to the "s"al ,atient
"n)er'oin' with)rawal@ i.e.@ a)e?"ate )oses o< benHo)iaHe,ines.
!he rare patient >ith status epilepticus m"st be treate)
a''ressively .
21N1(* )F 1LC2)2)L
71LC2)2)LIS-9
I'!()
Alcohol )e,en)ence is )e<ine) as re,eate) alcoholGrelate) )i<<ic"lties in
at least three o< seven areas o< <"nctionin' that cl"ster to'ether over a
*+Gmonth ,erio).
Alcohol ab"se is )e<ine) as re,etitive ,roblems with alcohol in any one
o< <o"r li<e areasNsocial@ inter,ersonal@ le'al@ an) occ",ational
Not everyone )evelo,s each o< the ,roblems )escribe) below .
'+(@)US S/S!+-
.NS [ BNS
1. a blac+out@ an e,iso)e o< tem,orary antero'ra)e amnesia@ in which the
,erson <or'ets all or ,art o< what occ"rre) )"rin' a )rin9in' evenin'.
+. )ist"rbe) slee,. sometimes )ist"rbin' )reams.
:. snorin' an) e3acerbate slee, a,nea
#. im,aire) I")'ment an) coor)ination@ increasin' the ris9 o< acci)ents an)
inI"ry
5. Heavy )rin9in' can also be associate) with hea)ache@ thirst@ na"sea@
vomitin'@ an) <ati'"e the <ollowin' )ay@ a han'over syn)rome that is
res,onsible <or si'ni<icant <inancial losses in most wor9 environments.
6. peripheral neuropathy
&. cerebellar )e'eneration or atro,hy.
8. 6ernic+e's (o,hthalmo,aresis@ ata3ia@ an) ence,halo,athy)
9. 7orsa+off's (retro'ra)e an) antero'ra)e amnesia) syndromes.
PS8%H9#:8
*. alcohol an)(or )r"' )e,en)ence.
2. schiHo,hrenia
:. manic )e,ressive )isease
TE>BORAR! BS.H1ATR1.
*. an3iety )isor)ers s"ch as ,anic )isor)er.
2. intense sadness lastin' <or )ays to wee9s
3. tem,orary severe an$iety
4. a")itory hallucinations
!he .astrointestinal System
Eso,ha'"s an) Stomach
*. e,i'astric )istress an) 'astrointestinal blee)in'.
+. hemorrha'ic 'astritis.
:. /iolent vomitin' can ,ro)"ce severe blee)in' thro"'h a >alloryG
Deiss lesion
Bancreas an) Liver
*. ac"te ,ancreatitis
+. alcoholGin)"ce) he,atitis@
:. cirrhosis
Cancer
*. breast cancer
+. oral an) eso,ha'eal cancers
:. rectal cancers
2ematopoietic System
*. 1< heavy )rin9in' is accom,anie) by <olic aci) )e<iciency@ there can
also be hy,erse'mente) ne"tro,hils@ retic"locyto,enia@ an) a
hy,er,lastic bone marrowE
+. i< maln"trition is ,resent@ si)eroblastic chan'es can be observe).
:. a ,ossible <alseGne'ative t"berc"lin s9in test
#. mil) thrombocyto,enia
Cardiovascular System
*. a )oseG)e,en)ent increase in bloo) ,ress"re
2. increase) ris9 <or coronary artery )isease
:. an increase) ris9 <or car)iomyo,athy.
#. >"ral thrombi can <orm in the le<t atri"m or ventricle
-. mitral re'"r'itation.
0. Atrial or ventric"lar arrhythmias@ es,ecially ,aro3ysmal tachycar)ia@
can also occ"r a<ter a )rin9in' bin'e in in)ivi)"als showin' no other
evi)ence o< heart )iseaseNa syn)rome 9nown as the Uholi)ay heart.U
This con)ition is observe) transiently in the maIority o< alcoholics
enterin' treatment.
.enitourinary System Changes= SeFual Functioning= and
Fetal *evelopment
>ALE
*. )ecrease erectile ca,acity in men.
+. irreversible testic"lar atro,hy with shrin9a'e o< the semini<ero"s t"b"les@
3. )ecreases in eIac"late vol"me@
4. a lower s,erm co"nt .
7E>ALE
*. amenorrhea@
+. in<ertility@
:. an increase) ris9 o< s,ontaneo"s abortion.
BRENAN.!
*. serio"s conse?"ences <or <etal )evelo,ment.
2. The fetal alcohol syndrome can incl")e any o< the <ollowin'R
<acial chan'es with e,icanthal eye <ol)sE
,oorly <orme) ear conchaE
small teeth with <a"lty enamelE
car)iac atrial or ventric"lar se,tal )e<ectsE
an aberrant ,almar crease
limitation in Ioint movementE
microce,haly with mental retar)ation..
-usculos<eletal
1. s9eletal m"scle wea9ness ca"se) by ac"te alcoholic myopathy
+. increase) ris9 <or <ract"res an) osteonecrosis o< the <emoral hea).
@+(!I.)
*+FI'I!I)'
an ill"sory or hall"cinatory sense o< movement o< the bo)y or
environment@ most o<ten a <eelin' o< s,innin'
P2/SI)L)./
Three sensory systems s"bservin' s,atial orientation an) ,ost"reE
*. The vestib"lar system is
+. the vis"al system (retina to occi,ital corte3)
:. the somatosensory system that conveys ,eri,heral in<ormation
<rom s9in@ Ioint@ an) m"scle rece,tors.
Physiologic @ertigo
This occ"rs in normal in)ivi)"als when
(*) the brain is con<ronte) with an intersensory mismatch amon' the
three stabiliHin' sensory systemsE
(+) the vestib"lar system is s"bIecte) to "n<amiliar hea) movements to
which it is "na)a,te)@ s"ch as in seasic9nessE
(:) "n"s"al hea)(nec9 ,ositions@ s"ch as the e3treme e3tension when
,aintin' a ceilin'E or <ollowin' a s,in.
Pathologic @ertigo
This res"lts <rom lesions o< the vis"al@ somatosensory@ or vestib"lar
systems.
/is"al verti'o
*. ca"se) by new or incorrect eye'lasses or by the s"))en onset o< an
e3traoc"lar m"scle ,aresis with )i,lo,iaE
Somatosensory verti'o@
*. rare in isolation@
+. "s"ally )"e to a ,eri,heral ne"ro,athy or myelo,athy
vestib"lar verti'o
*. The most common ca"se o< ,atholo'ic verti'o
+. involvin' either its en) or'an (labyrinth)@ nerve@ or central
connections.
:. The verti'o is associate) with Ier9 nysta'm"s an) is <re?"ently
accom,anie) by na"sea@ ,ost"ral "nstea)iness@ an) 'ait ata3ia.
#. Since verti'o increases with ra,i) hea) movements@ ,atients
ten) to hol) their hea)s still.
Labyrinthine *ysfunction
This ca"ses severe rotational or linear verti'o.
The <ast ,hases o< nysta'm"s beat away <rom the lesion si)e@
the ten)ency to <all is towar) the si)e o< the lesion@ ,artic"larly in
)ar9ness or with the eyes close).
1cute unilateral labyrinthine dysfunction
*. 1n<ectionG her,es sim,le3 vir"s ty,e *@
+. tra"ma@
:. ischemia.
o ,res"mably )"e to occl"sion o< the labyrinthine branch o< the
internal a")itory artery@ may be the sole mani<estation o<
vertebrobasilar ins"<<iciency E
1cute bilateral labyrinthine dysfunction
o "s"ally the res"lt o< to3ins s"ch as )r"'s or alcohol.
o The most common o<<en)in' )r"'s are the amino'lycosi)e
antibiotics
(ecurrent unilateral labyrinthine dysfunction =
o in association with si'ns an) sym,toms o< cochlear )isease
(,ro'ressive hearin' loss an) tinnit"s)@ is "s"ally )"e to >\ni_reFs
)isease
Positional vertigo
o ,reci,itate) by a rec"mbent hea) ,osition@ either to the ri'ht or to
the le<t.
o 4eni'n ,aro3ysmal ,ositional (or ,ositionin') verti'o (4BB/) o< the
,osterior semicirc"lar canal is ,artic"larly common.
o Altho"'h the con)ition may be )"e to hea) tra"ma@ "s"ally no
,reci,itatin' <actors are i)enti<ie).
4eni'n Baro3ysmal Bositional /erti'o an) .entral Bositional /erti'o
Features 5PP@ Central
*. Latency

:%#$ s NoneR imme)iate verti'o an)
nysta'm"s
+. 7ati'ability

!es No
:. Habit"ation

!es No
#. 1ntensity o<
verti'o
Severe >il)
-. Re,ro)"cibilit
y

/ariable oo)
1 perilymphatic fistula
o sho"l) be s"s,ecte) when e,iso)ic verti'o is ,reci,itate) by
/alsalva or e3ertion
o The con)ition is "s"ally ca"se) by hea) tra"ma or barotra"ma or
occ"rs a<ter mi))le ear s"r'ery.
@ertigo of @estibular 'erve )rigin
The most common ca"se o< ei'hth cranial nerve )ys<"nction is a t"mor@
"s"ally a schwannoma (acoustic neuroma) or a menin'ioma.
a")itory sym,toms are the most common mani<estations.
Central @ertigo
Lesions o< the brainstem or cerebell"m can ca"se ac"te verti'o@ b"t
associate) si'ns an) sym,toms "s"ally ,ermit )istinction <rom a
labyrinthine etiolo'y
7eat"res o< Beri,heral an) .entral /erti'o
Sign or Symptom Peripheral 7Labyrinth9 Central 75rainstem or
Cerebellum9
*. ;irection o<
associate)
nysta'm"s
Uni)irectionalE <ast ,hase
o,,osite lesion
4i)irectional or
"ni)irectional
+. B"rely
horiHontal
nysta'm"s
Uncommon .ommon
:. /ertical
nysta'm"s
Never ,resent >ay be ,resent
#. Severity o<
verti'o
>ar9e) O<ten mil)
-. ;irection o<
<all
Towar) slow ,hase /ariable
0. ;"ration o<
sym,toms
7inite (min"tes@ )ays@
wee9s) b"t rec"rrent
>ay be chronic
&. Tinnit"s
an)(or )ea<ness
O<ten ,resent Us"ally absent
6. Associate)
.NS
abnormalities
None E3tremely common (e.'.@
)i,lo,ia@ hicc",s@ cranial
ne"ro,athies@ )ysarthria)
2. .ommon
ca"ses
4BB/@ in<ection
(labyrinthitis)@ >\ni_reFs@
ne"ronitis@ ischemia@
tra"ma@ to3in
/asc"lar@ )emyelinatin'@
neo,lasm
mi'raine a"ra

;estibular epilepsy@ verti'o secon)ary to tem,oral lobe e,ile,tic activity@
is rare.

Bsycho'enic /erti'o
o 1t sho"l) be s"s,ecte) in ,atients so Uinca,acitate)U by their sym,toms
that they a)o,t a ,rolon'e) ho"sebo"n) stat"s.
o a ,sycho'enic etiolo'y is almost certain when nysta'm"s is absent
)"rin' a verti'ino"s e,iso)e.
*I1.')S!IC +@1LU1!I)'
The sim,lest ,rovocative test <or vestib"lar )ys<"nction is ra,i) rotation
an) abr",t cessation o< movement in a swivel chair.
Batients with sym,toms o< ,ositional verti'o sho"l) be a,,ro,riately
teste) .
A <inal ,rovocative an) )ia'nostic vestib"lar test@ re?"irin' the "se o<
7renHel eye'lasses@ is vi'oro"s hea) sha9in' in the horiHontal ,lane <or
abo"t *$ s. 1< nysta'm"s )evelo,s a<ter the sha9in' sto,s@ even in the
absence o< verti'o@ vestib"lar )ys<"nction is )emonstrate). The
mane"ver can then be re,eate) in the vertical ,lane.
1< the ,rovocative tests establish the )iHHiness as a vestib"lar sym,tom@
an eval"ation o< vestib"lar verti'o is "n)erta9en
-1'1.+-+'!
Treatment o< ac"te verti'o consists o< be) rest (*%+ )ays ma3im"m) an)
vestib"lar s",,ressant )r"'s
Treatment o< /erti'o
1gent
a

*. Antihistamines
>ecliHine
BromethaHine
c c
7or ac"te verti'o only
+. 4enHo)iaHe,ines
;iaHe,am
.lonaHe,am
:. BhenothiaHines
Brochlor,eraHine
c c
7or ac"te verti'o only
1gent
a


#. Anticholiner'ic
d

d
7or motion sic9ness only.
Sco,olamine trans)ermal
-. Sym,athomimetics
d

d
7or motion sic9ness only.
E,he)rine
0. .ombination
,re,arations
d

d
7or motion sic9ness only.
E,he)rine an)
,romethaHine
&. E3ercise thera,y
Re,ositionin'
mane"vers
e

7or beni'n ,aro3ysmal ,ositional verti'o.
/estib"lar rehabilitation
f


6. Other
;i"retics or lowGsalt (* '())
)iet
g

7or >\ni_reFs )isease.
Antimi'raino"s )r"'s
h


h
7or mi'raineGassociate) verti'o
1nner ear s"r'ery
i


i
7or ,erilym,hatic <ist"la an) re<ractory cases o<
>\ni_reFs )isease.
l"cocorticoi)s
c c
7or ac"te verti'o only
.
S/'C)P+
*+FI'I!I)' O I'!()
Syncope@ a transient loss of consciousness and postural tone )"e
to re)"ce) cerebral bloo) <low@ is associate) with spontaneous
recovery.
1t may occ"r suddenly= >ithout >arning@ or may be ,rece)e) by
symptoms of faintness 7LpresyncopeL9&
These sym,toms include
*. li'hthea)e)ness@
+. )iHHiness@
:. a <eelin' o< warmth@
#. )ia,horesis@
-. na"sea@ an)
0. vis"al bl"rrin'
&. occasionally ,rocee)in' to transient blin)ness.
Synco,e may be benign when it occ"rs as a res"lt o< normal
car)iovasc"lar re<le3 e<<ects on heart rate an) vasc"lar tone@ or serious
when )"e to a li<eGthreatenin' car)iac arrhythmia.
Synco,e may occ"r as a single event or may be recurrent.
(ecurrent= uneFplained syncope= particularly in an individual
>ith structural heart disease@ is associate) with a hi'h ris9 o< )eath
(#$= mortality within + years)
C1US+S
1. ;isor)ers o< /asc"lar Tone or 4loo) /ol"me
A. Re<le3 synco,es
*. Ne"rocar)io'enic
+. Sit"ational
o .o"'h
o >ict"rition
o ;e<ecation
o /alsalva
o ;e'l"tition
:. .aroti) sin"s hy,ersensitivity
4. Orthostatic hy,otension
*. ;r"'Gin)"ce) (antihy,ertensive or vaso)ilator )r"'s)
+. B"re a"tonomic <ail"re (i)io,athic orthostatic hy,otension)
:. >"ltisystem atro,hies
#. Beri,heral ne"ro,athy ()iabetic@ alcoholic@ n"tritional@ amyloi))
-. Bhysical )econ)itionin'
0. Sym,athectomy
&. ;ecrease) bloo) vol"me
11. .ar)iovasc"lar ;isor)ers
A. Str"ct"ral an) obstr"ctive ca"ses
*. B"lmonary embolism
+. B"lmonary hy,ertension
:. Atrial my3oma
#. >itral valv"lar stenosis
-. >yocar)ial )isease (massive ac"te myocar)ial in<arction)
0. Le<t ventric"lar myocar)ial restriction or constriction
&. Bericar)ial constriction or tam,ona)e
6. Aortic o"t<low tract obstr"ction
2. Aortic valv"lar stenosis
*$. Hy,ertro,hic obstr"ctive car)iomyo,athy
4. .ar)iac arrhythmias
*. 4ra)yarrhythmias
a. Sin"s bra)ycar)ia@ sinoatrial bloc9@ sin"s arrest@ sic9Gsin"s
syn)rome
b. Atrioventric"lar bloc9
+. Tachyarrhythmias
a. S",raventric"lar tachycar)ia with str"ct"ral car)iovasc"lar
)isease
b. Atrial <ibrillation with the Dol<<GBar9insonGDhite syn)rome
c. Atrial <l"tter with *R* atrioventric"lar con)"ction
). /entric"lar tachycar)ia
111. .erebrovasc"lar ;isease
A. /ertebrobasilar ins"<<iciency
4. 4asilar artery mi'raine
1/. Other ;isor)ers that >ay Resemble Synco,e
A. >etabolic
*. Hy,o3ia
+. Anemia
:. ;iminishe) carbon )io3i)e )"e to hy,erventilation
#. Hy,o'lycemia
4. Bsycho'enic
*. An3iety attac9s
+. Hysterical <aintin'
.. SeiH"res
*IFF+(+'!I1L *I1.')SIS
3 AnAiet% Attac?s and /%'er!entilati"n S%ndr"#e
the sym,toms are not accompanied by facial pallor an) are not
relieved by recumbency.
4 SeiBures
7eat"res that ;istin'"ish eneraliHe) TonicG.lonic SeiH"re <rom Synco,e
Features SeiCure Syncope
*. 1mme)iate
,reci,itatin' <actors
Us"ally none Emotional stress@ /alsalva@
orthostatic hy,otension@
car)iac etiolo'ies
+. Bremonitory
sym,toms
None or a"ra
(e.'.@ o)) o)or)
Tire)ness@ na"sea@
)ia,horesis@ t"nnelin' o< vision
:. Bost"re at onset /ariable Us"ally erect
0& !ransition to
unconsciousness
)ften
immediate
.radual over seconds
#& *uration of
unconsciousness
-inutes Seconds
3& *uration of tonic
or clonic movements
,0$30 s 'ever more than 1# s
4& Facial appearance
during event
Cyanosis=
frothing at
mouth
Pallor
6. ;isorientation an)
slee,iness a<ter event
>any min"tes to
ho"rs
5- min
2. Achin' o< m"scles
a<ter event
O<ten Sometimes
*$. 4itin' o< ton'"e Sometimes Rarely
**. 1ncontinence Sometimes Sometimes
*+. Hea)ache Sometimes Rarely
5 /%'"gl%ce#ia
6 /%sterical Fainting
Lac< of change in pulse and blood pressure or color of the s<in
and mucous membranes )istin'"ish it <rom the vaso)e,ressor <aint.
!(+1!-+'!
SI!+ )F C1(+
Patients >ith syncope should be hospitaliCed >hen there is a
,ossibility that the e,iso)e may have res"lte) <rom a li<eGthreatenin'
abnormality or i< rec"rrence with si'ni<icant inI"ry seems li9ely.
These in)ivi)"als sho"l) be a)mitte) to a be) with continuous
electrocardiographic monitoring.
Patients >ho are <no>n to have a normal heart and for >hom
the history strongly suggests vasovagal or situational syncope
may be treate) as o"t,atients i< the e,iso)es are neither <re?"ent nor
severe.
.+'+(1L
Certain precautions should be ta<en regardless of the cause of
syncope&
?21! ?ILL P1!I+'! *) P
Patients >ith freHuent episodes= or those >ho have eFperienced
syncope >ithout >arning symptoms@ sho"l) avoi) sit"ations in
which s"))en loss o< conscio"sness mi'ht res"lt in inI"ry (e.'.@ climbin'
la))ers@ swimmin' alone@ o,eratin' heavy machinery@ )rivin').
1t the first sign of symptoms@ ,atients sho"l) ma9e every e<<ort to
avoi) inI"ry .
Patients should lower their hea) to the e3tent ,ossible an) ,re<erably
sho"l) lie )own.
Lo>ering the head by bending at the >aist should be avoided
beca"se it may <"rther com,romise veno"s ret"rn to the heart.
()L+ )F (+L1!I@+ O F(I+'*S
Dhen a,,ro,riate@ family members or other close contacts should
be educated as to the ,roblem.
!his >ill ensure appropriate therapy and may prevent delivery
of inappropriate therapy (chest com,ressions associate) with
car)io,"lmonary res"scitation) that may in<lict tra"ma.
P! ?I!2 L)S! C)CI)US'+SS
*. Patients >ho have lost consciousness sho"l) be ,lace) in a ,osition
that ma3imiHes cerebral bloo) <low@ o<<ers ,rotection <rom tra"ma@ an)
sec"res the airway.
+. Dhenever ,ossible@ the ,atient sho"l) be placed supine >ith the head
turned to the side to ,revent as,iration an) the ton'"e <rom bloc9in'
the airway.
:. 1ssessment of the pulse and direct cardiac auscultation may assist
in )eterminin' i< the e,iso)e is associate) with a bra)yarrhythmia or a
tachyarrhythmia.
#. Clothing that fits tightly around the nec< or waist sho"l) be
loosene).
-. Peripheral stimulation@ s"ch as s,rin9lin' col) water on the <ace@ may
be hel,<"l.
0. Patients should not be given anything by mouth or be permitted to
rise until the sense of physical >ea<ness has passed&
SP+CIFIC
The treatment o< synco,e is directed at the underlying cause&
Patients >ith vasovagal syncope
Patients >ith vasovagal syncope sho"l) be instr"cte) to avoi)
sit"ations or stim"li that have ca"se) them to lose conscio"sness an) to
ass"me a rec"mbent ,osition when ,remonitory sym,toms occ"r.
!hese behavioral modifications alone may be sufficient <or
,atients with in<re?"ent an) relatively beni'n e,iso)es o< vasova'al
synco,e@ ,artic"larly when loss o< conscio"sness occ"rs in res,onse to a
s,eci<ic stim"l"s.
+pisodes associated >ith intravascular volume depletion may be
,revente) by salt an) <l"i) loa)in' ,rior to ,rovocative events.
*rug therapy may be necessary when vasova'al synco,e is resistant
to the above meas"res@ when e,iso)es occ"r <re?"ently@ or when
synco,e is associate) with a si'ni<icant ris9 <or inI"ry.
5+!1A1drenergic receptor antagonists (metoprolol@ +-%-$ m' bi)E
atenolol= +-%-$ m' ?)E or nadolol@ *$%+$ m' bi)E all startin' )oses)@
the most wi)ely "se) a'ents
Serotonin reupta<e inhibitors (paroFetine@ +$%#$ m' ?)E or
sertraline@ +-%-$ m' ?))@ a,,ear to be e<<ective <or some ,atients.
5upropion SR (*-$ m' ?))@ another anti)e,ressant@ has also been "se)
with s"ccess.
4ETAGA)rener'ic rece,tor anta'onists an) serotonin re",ta9e inhibitors
are >ell tolerated and are often used as firstAline agents for
younger patients.
2ydrofludrocortisone ($.*%$.+ m' ?))@ a mineralocorticoid=
,romotes so)i"m retention@ vol"me e3,ansion@ an) ,eri,heral
vasoconstriction by increasin' 4ETAGrece,tor sensitivity to en)o'eno"s
catecholamines.
Hy)ro<l")rocortisone is "se<"l <or patients >ith intravascular volume
depletion and for those >ho also have postural hypotension&
Proamatine (+.-%*$ m' bi) or ti))@ an 1LP21Aagonist@ has been "se)
as a <irstGline a'ent <or some ,atients.
2o>ever= in some patients= proamatine and
hydrofludrocortisone may increase resting supine systemic
blood pressure= which may be ,roblematic <or those with hy,ertension.
*isopyramide (*-$ m' bi))@ a vagolytic antiarrhythmic )r"' with
ne'ative inotro,ic ,ro,erties@ an) trans)ermal scopolamine= another
vagolytic@ have been "se) to treat vasova'al synco,e@ as have
theophylline and ephedrine.
Si)e e<<ects associate) with these )r"'s have limite) their "se <or this
in)ication.
*ualAchamber cardiac pacing may be e<<ective <or ,atients with
<re?"ent e,iso)es o< vasova'al synco,e@ ,artic"larly <or those with
,rolon'e) asystole associate) with vasova'al e,iso)es.
Pacema<ers that can be programmed to transiently pace at a
high rate 760$100 beatsmin9 a<ter a ,ro<o"n) )ro, in the ,atientFs
intrinsic heart rate are most e<<ective.
Patients >ith orthostatic hypotension
Patients >ith orthostatic hypotension sho"l) be instr"cte) to rise
slowly an) systematically (s",ine to seate)@ seate) to stan)in') <rom the
be) or a chair.
>ovement o< the le's ,rior to risin' <acilitates veno"s ret"rn <rom the
lower e3tremities.
Dhenever ,ossible@ medications that aggravate the problem
7vasodilators= diuretics= etc.) sho"l) be )iscontin"e).
+levation of the head of the bed Q20$,0 cm 7"$12 in&9R and use of
compression stoc<ings may hel,.
1dditional therapeutic modalities include salt loa)in' an) a variety
o< ,harmacolo'ic a'ents incl")in'
*. sym,athomimetic amines@
+. monamine o3i)ase inhibitors@
:. beta bloc9ers@
#. levo)o,a.
.lossopharyngeal neuralgia
.lossopharyngeal neuralgia is treate) with carbamaCepine@ which
is e<<ective <or synco,e as well as <or ,ain.
carotid sinus hypersensitivity
Patients >ith carotid sinus hypersensitivity sho"l) be instr"cte) to
avoi) clothin' an) sit"ations that stim"late caroti) sin"s barorece,tors.
They sho"l) t"rn their entire bo)y@ rather than I"st their hea)@ when
loo9in' to the si)e.
!hose >ith intractable syncope due to the cardioinhibitory response
to carotid sinus stimulation
sho"l) "n)er'o ,ermanent ,acema9er im,lantation.
N"nc"#'ressi!e
M%el"'aties
C1US+S
The most <re?"ent ca"ses o< noncom,ressive ac"te transverse
myelo,athy (AT>) are
*. s,inal cor) in<arctionE
+. systemic in<lammatory )isor)ers@ incl")in' SLE an)
sarcoi)osisE
:. )emyelinatin' )iseases@ incl")in' m"lti,le sclerosis (>S)E
#. ,ostin<ectio"s or i)io,athic transverse myelitis@
-. in<ectio"s (,rimarily viral) .
I'@+S!I.1!I)'S
A<ter s,inal cor) com,ression is e3cl")e)@ the eval"ation
'enerally re?"ires a l"mbar ,"nct"re an) a search <or "n)erlyin'
systemic )isease
Eval"ation
*. >R1 o< s,inal cor) with an) witho"t contrast
e3cl")e com,ressive ca"ses.
+. .S7 st")iesR
.ell co"nt@ ,rotein@ 'l"cose@ 1' in)e3(synthesis rate@
oli'oclonal ban)s@
/;RLE
ramFs stain@ aci)G<ast bacilli@ an) 1n)ia in9 stainsE
B.R <or /`/@ HS/G+@ HS/G*@ E4/@ .>/@ H1/E
antibo)y <or HTL/G1@ >. ,ne"moniae@ an) .hlamy)ia
,ne"moniaeE
viral@ bacterial@ mycobacterial@ an) <"n'al c"lt"res.
:. 4loo) st")ies <or in<ectionR
H1/E
1' an) 1'> enterovir"s antibo)yE
1'> m"m,s@ measles@ r"bella@
#. 1mm"neGme)iate) )isor)ersR
ESRE
ANAE )s;NAE
rhe"matoi) <actorE
anti,hos,holi,i) an) anticar)ioli,in antibo)iesE
-. Sarcoi)osisR
Ser"m an'iotensinGconvertin' enHymeE
ser"m .aE
+# ho"r "rine .aE
chest 3GrayE
chest .TE
0. ;emyelinatin' )iseaseR
4rain >R1 scan@
evo9e) ,otentials@
.S7 oli'oclonal ban)s@
&. /asc"lar ca"sesR
.T myelo'ramE
s,inal an'io'ram.
CLI'IC1L F+1!U(+ O 5(I+F -1'1.+-+'!
S,inal .or) 1n<arction
Ac"te in<arction in the territory o< the anterior s,inal artery
,ro)"ces
*) ,ara,le'ia or ?"a)ri,le'ia@
+) )issociate) sensory loss a<<ectin' ,ain an) tem,erat"re sense
b"t s,arin' vibration an) ,osition sense@ an) loss o< s,hincter
control (Uanterior cor) syn)romeU).
:) Onset may be s"))en an) )ramatic b"t more ty,ically is
,ro'ressive over min"tes or a <ew ho"rs.
#) Shar, mi)line or ra)iatin' bac9 ,ain localiHe) to the area o<
ischemia is <re?"ent.
-) Are<le3ia )"e to s,inal shoc9 is o<ten ,resent initiallyE with
time@ hy,erre<le3ia an) s,asticity a,,ear.
0) Less common is in<arction in the territory o< the ,osterior s,inal
arteries@ res"ltin' in loss o< ,osterior col"mn <"nction.
The anti,hos,holi,i) antibo)y syn)rome is treate) with
anticoa'"lation.
;raina'e o< s,inal <l"i) has re,orte)ly been s"ccess<"l in some
cases o< cor) in<arction
1n<lammatory an) 1mm"ne >yelo,athies (>yelitis)
Systemic 1n<lammatory ;isor)ers
*) SLE . Res,onses to 'l"cocorticoi)s an)(or cyclo,hos,hami)e
+) SIW'renFs syn)rome@
:) 4ehJetFs syn)rome@
#) /asc"litis
-) sarcoi) myelo,athy G1nitial treatment is with oral
'l"cocorticoi)sE imm"nos",,ressant )r"'s are "se) <or
resistant cases.
;emyelinatin' >yelo,athies
*) >"lti,le sclerosis
+) Ne"romyelitis o,tica (N>O)
o 1ntraveno"s methyl,re)nisolone (-$$ m' ?) <or : )ays)
<ollowe) by oral ,re)nisone (* m'(9' ,er )ay <or several
wee9s@ then 'ra)"al ta,er) has been "se) as initial
treatment.
o A co"rse o< ,lasma e3chan'e is in)icate) <or severe cases
i< 'l"cocorticoi)s are ine<<ective.
Bostin<ectio"s >yelitis
*) >any cases o< myelitis@ terme) ,ostin<ectio"s or ,ostvaccinal@
<ollow an in<ection or vaccination.
+) E,steinG4arr vir"s (E4/)@
:) cytome'alovir"s (.>/)@
#) myco,lasma@
-) in<l"enHa@
0) measles@
&) varicella@
6) r"beola@
2) m"m,s.
Ac"te 1n<ectio"s >yelitis
*) Her,es Hoster
+) HS/ ty,es * an) +@
:) E4/@
#) .>/@
-) rabies vir"s
0) mycobacterial myelitis (most are essentially abscesses)
&) Listeria monocyto'enes@
6) Lyme )isease
2) Sy,hilis
*$) Schistosomiasis
**) To3o,lasmosis
o Her,es Hoster@ HS/@ an) E4/ myelitis are treate) with
intraveno"s acyclovir (*$ m'(9' ?6h) or oral valacyclovir (+
'm ti)) <or *$%*# )aysE
o .>/ with 'anciclovir (- m'(9' 1/ bi)) ,l"s <oscarnet (0$
m'(9' 1/ ti))@ or ci)o<ovir (- m'(9' ,er wee9 <or + wee9s).

You might also like